You are on page 1of 112

DIVISION LAS DEVELOPMENT TEAM FOR SENIOR

HIGH SCHOOL – STATISTICS AND PROBABILITY

Norma B. Samantela, CESO VI Schools Division Superintendent


Ma. Jeany T. Abayon, Ed.D. Assistant Schools Division Superintendent
Lauro B. Millano, CESE Assistant Schools Division Superintendent

DIVISION EVALUATION TEAM


CID Chief, Dr. Sancita B. Peñarubia

PSDS/Division Mathematics Coordinator, Dr. Felma A. Bonito

WRITERS:

Quarter 3 Quarter 4
Catherine R. Realingo Catherine R. Realingo
Marijo S. Balin Marijo S. Balin
Amie-Lyn S. Cedo Amie-Lyn S. Cedo
Charmie G. Rellores Charmie G. Rellores
Aris B. Nisola Aris B. Nisola
Margie C. Macatingrao Margie C. Macatingrao
Giovanni G. Naag Giovanni G. Naag
Albert S. Corbilla Albert S. Corbilla
Hilda C. Remendado Hilda C. Remendado
Mary Francia S. Rico Mary Francia S. Rico

Team Leader : Elenita L. Nocidal


Content Editors : Elenita L. Nocidal
Jocelyn P. Castelo
Cover Layout Artist : Albert S. Corbilla
IT : Joseph C. Bueno
CONTENT
Week No. Learning Competencies Page
Illustrate a random variable (M11/12SP-IIIa-1) No.
Distinguish between a discrete and a continuous
Week 1 random variable (M11/12SP-IIIa-2) 1
Find the possible values of a random variable
(M11/12SP-IIIa-3)
Illustrates a probability distribution for a discrete
random variable and its properties. (M11/12SP-
IIIa-4)
Constructs the probability mass function of a
discrete random variable and its corresponding 5
histogram. (M11/12SP-IIIa-5)
Computes probabilities corresponding to a given
random variable. (M11/12SP-IIIa-6)
Week 2 Illustrates the mean of a discrete random variable.
M11/12SP-IIIb-1
Calculate the mean of a discrete random variable.
M11/12SP-IIIb-2
15
Interprets the mean of a discrete random variable.
M11/12SP-IIIb-3

Solves problem involving mean of a discrete probability


distribution. M11/12SP-IIIb-4
Calculate the Variance of a Discrete Random
Variable (M11/12SP-111b-2)
Interprets the Variance of a Discrete Random Variable
18
(M11/12SP-111b-3)
Week 3 Solve problems involving Variance of probability
distribution (M11/12SP-IIIb-4)
Illustrate normal random variable and its characteristics
(M11/12SP-IIIc-1) 25
Identifies region under the normal curve
Week 4 corresponding to different standard normal vales. 31
(M11/12SP-111C3)
Convert a normal random variable to a standard normal
variable and vice versa (MII/12SP-IIIc4) 39
Compute probabilities and percentiles using
standard normal table 45
(MII/12SP-IIIc-d-1)
The learner illustrates random sampling.
(M11/12SP-IIIc-2) 50
The learner distinguishes between parameter and statistic.
Week 5 56
(M11/12SP-IIIc-3)
The learner identifies sampling distributions of
statistics (sample mean) (M11/12SP-IIIc-4) 60
Find the Mean and Variance of the Sampling Distribution
of the Sample Mean (M11/12SP-IIId-5)
Define the sampling distribution of the sample mean for 67
Week 6
normal population when the variance is (a) known; and
(b) unknown (M11/12SP-IIIe-1)

Illustrates the Central Limit Theorem (M11/12SP-


IIIe-2)
78
Defines the sampling distribution of the sample mean
Week 7 to 8 using the Central Limit Theorem (M11/12SP- III-3)
Solve problems involving sampling distributions of the
sample mean. ( M11SP/IIIe-f-1) 83
The learner illustrates t- distribution (M11/12SP-
IIIg-2) 88
Week 9
Identifying percentiles using t-distribution
(M11/12SP-IIIg-5) 94
Identifies the length of a confidence interval: LC Code
(M11/12SP-IIIj-1)
96
Computes for the length of confidence interval: LC Code
(M11/12SP-IIIj-1)

Week 10 Computes for an appropriate sample size using the


length of the confidence interval. (M11/12SP-IIIj-3)
Solve problems involving sample size determination. 103
(M11/12SP-IIIj-4)
STATISTICS AND PROBABILITY

Name of Learner: _____________________________ Section: _________________


Grade Level: _________________________________ Date: ___________________

Q3_W1_LAS1
EXPLORING RANDOM VARIABLES

I. BACKGROUND INFORMATION FOR LEARNERS

You have learned in your past lessons in mathematics that an experiment is any activity, which
can be done repeatedly under similar conditions. The set of all possible outcomes of an experiment is
called the sample space. You have also learned how to systematically list the possible outcomes of a
given experiment.
Let us recall, the finding the sample space of an experiment.
• In throwing a die once, the sample space is S = {1, 2, 3, 4, 5, 6}.
• The sample space of rolling a die and tossing a coin simultaneously is,
S = 1H ,1T ,2 H ,2T ,3H ,3T ,4 H ,4T ,5H ,5T ,6 H ,6T 
• The sample space of tossing three coins is
S= {TTT, TTH, THT, HTT, HHT, HTH, THH, HHH}.
This time this activity sheet will explain to us the concept of random variables, find the possible
values of it and its classification.
A random variable is a set of possible values for a random experiment. It is denoted by a
capital letter, usually X, Y and Z.
Some examples of random experiments are:
• tossing a coin three times,
• rolling a die twice,
• drawing two balls from a box.
Let us try to illustrate and give the value of random variables using the following:
Example 1:
In tossing a coin, let X be the random variable denoting the number of tails. Find the
value of the random variable X.
Solution:
Random variable Possible Outcomes Possible values

HEAD O
X = no. of tails
TAIL 1

So, the possible values of the random variable X are 0 and 1.


Example 2:
A coin is tossed 3 times. Let X be the random variable denoting the number of
heads. Find the values of the random variable X .
Solution: To do this, identify all the possible outcomes of the given statistical
experiment. Determine the specific random variable defined in the problem i.e.
Outcomes TTT TTH THT THH HTT HTH HHT HHH

𝑥 = 𝑛𝑜. 𝑜𝑓 ℎ𝑒𝑎𝑑𝑠 0 1 1 2 1 2 2 3

Therefore, the possible values of the random variable X are 0, 1, 2 & 3.

1
Example 3:
The number of even number outcomes in a roll of a die. Let Y be the random
variable denoting the number of even numbers. Write the possible values of Y.
Solution:
Outcomes = since we are looking for even numbers, therefore,
the possible values of the random variable X are , & , that is the possible
values of X are 2, 4, and 6.

Types of random variables


• Discrete random variable is a variable whose value is obtained by counting.
Examples:
1. The number of students in a class
2. The number of test questions answered correctly.
3. The results or outcomes of rolling 2 dice.

• Continuous random variable is a variable whose value is obtained by measuring.


Examples:
1. The amount of time required to complete a project
2. The amount of rain falls in a storm
3. The height of children
4. The amount of time it takes to sell shoes

II. LEARNING COMPETENCIES:


1. Illustrates a random variable (M11/12SP-IIIa-1)
2. Distinguishes between a discrete and a continuous random variable (M11/12SP-IIIa-2)
3. Finds the possible values of a random variable (M11/12SP-IIIa-3)

III. ACTIVITIES:

Activity 1 CLASSIFY ME!

Classify the following random variables as discrete or continuous.


____________ 1. The number of defective computers produced by a manufacturer
____________ 2. The weight of newborn in a hospital
____________ 3. The number of siblings in a family
____________ 4. The amount of paint utilized for a classroom
____________ 5. The number of dropouts in a school district for a period of 10 years
____________ 6. The speed of a car
____________ 7. The numbers of female athletes of the Sports track
____________ 8. The time needed to finish the test
____________ 9. The amount of sugar in a cup of coffee
____________10. The number of people who are playing LOTTO each day

THINK & REFLECT:


How do you know whether a random variable is continuous or discrete?
______________________________________________________________________________
What is the difference between continuous and discrete random variables?
_______________________________________________________________________________

2
Activity 2 RE-VISIT & RECALL

Directions: List the sample space of the following experiments.

Experiment Sample Space

1. Tossing two coins

2. Getting a sum of 5 when a pair of dice are rolled

3. Drawing a diamond from a deck of cards

4. Drawing two randomly selected items from a box


of two defective and three non-defective items.

THINK & REFLECT:


How could you determine the possible outcomes/sample space of the following experiment?
___________________________________________________________________________

Activity 3 SOLVE ME!

Illustrate each problem using table values and solve.


1. Suppose three cell phones are tested at random where D represent the defective cell phone and N
represent the non-defective cell phone. Let X represent the number of defective cell phones that
occur, show the values of the random variable X .
2. Two balls are drawn in succession without replacement from an urn containing 5 red balls and 6
blue balls. Let Z be the random variable representing the number of blue balls. Find the values of
the random variable Z .
3. Four coins are tossed. Let Y be the random variable representing the number of heads that occur.
Find the values of the random variable Y .
4. A shipment of five computers contains units that are defective. If a retailer receives three of these
computers at random, list the elements of the sample space S using the letters D and N for defective
and non-defective computers, respectively. To each sample point, assign a value x of the random
variable X representing the number of computers purchased by the retailer which are defective.
5. A family has four children. If Y is a random variable that pertains to the number of female children,
what are the possible values of Y?

THINK & REFLECT:


How do you find the value of random variables?
___________________________________________________________________________
__________________________________________________________________________________
__________________________________________________________________________________
Enumerate the steps.
___________________________________________________________________________
__________________________________________________________________________________
_________________________________________________________________________________

3
4
Writer
CATHERINE R. REALINGO
Prepared by:
Answer Key.
ACTIVITY#1. Classify Me!
1. Discrete 2. Continuous 3. Discrete 4. Continuous 5. Discrete
6. Continuous 7. Discrete 8. Continuous 9. Continuous 10. Discrete
ACTIVITY #2. Revisit & Recall
1. S = {HH, HT, TH, TT} 2. S = {1,4),(4,1),(2,3),(3,2)} 3. S = {1, 2, 3, 4, 5, 6, 7, 8, 9, 10, J, Q, K}
4. S = {DD,ND,DN,NN}
ACTIVITY#3. Solve Me!
1.
Possible outcomes NNN NND NDN DNN NDD DND DDN DDD
X=the number of defective cell phones O 1 1 1 2 2 2 3
Therefore, the values of the random variable X are 0, 1, 2, and 3.
2.
Possible outcomes RR RB BR BB
Z = the number blue balls 0 1 1 2
Therefore, the values of the random variable Z are 0, 1, and 2.
3.
Possible TTT TTT TT TH HT TT TH HT HH THH HHH HTH HHT HHH
outcomes T H HT TT TT HH TH HT TT H T H H H
Y=The 0 1 1 1 1 2 2 2 2 3 3 3 3 4
number of
heads
Therefore, the values of the random variable Y are 0, 1, 2, 3, and 4.
4.
Possible outcomes NNN NND NDN DNN NDD DND DDN DDD
X=the number of defectives computer O 1 1 1 2 2 2 3
Therefore, the values of the random variable X are 0, 1, 2 and 3.
5.
Possible MM MM MMF MF FM MM MF FM FF MF FM FF FFF FF
outcome MM MF M MM MM FF MF FM MM FF FF MF M FF
Y=the 0 1 1 1 1 2 2 2 2 3 3 3 3 4
number
of
females
Therefore, the values of the random variable Yare 0, 1, 2, 3, and 4
Distributions, First Edition, 2020
Monina C. Raagas, Mardy Nelle V. Galve, Module 1: Random Variable & Probability
Probability (Grade 11)Q3–
Daisy Rodriquez Acas, Probability Distribution of Random Variables (Modules Statistics &
Rene R. Belecina, Elisa S. Baccay et. al, Statistics and Probability-First Edition
References:
STATISTICS AND PROBABILITY

Name of Learner: _____________________________ Section: _________________


Grade Level: _________________________________ Date: ___________________

Q3_W2_LAS2
CONSTRUCTING PROBABILITY DISTRIBUTIONS

I. BACKGROUND INFORMATION FOR LEARNERS

Welcome back dear young mathematician! You are now on second week of Quarter 3.
In the previous week you were able to illustrate a random variable, classify them as discrete or
continuous; and find the possible values of it.

This week, we will be exploring Probability Distributions of Discrete Random Variable.


After knowing the possible outcomes, you can compute for the corresponding probabilities,
construct probability distribution, and draw a histogram (bar graph).

Learning this topic will probably help you in decision-making. Decision-making is an


important aspect in business, education, insurance, other real-life situations. Many decisions
are made by assigning probabilities to all possible outcomes pertaining to the situation and then
evaluating the results. For instance, an insurance company might be able to assign probabilities
to the number of vehicles a family owns. This information will help the company in making
decisions regarding future financial situations. This situation requires the use of random
variables and probability distribution.

Discrete Probability Distribution


Now, it’s time for you to explore and learn. Good luck young mathematician!
A Discrete Probability Distribution or a Probability Mass Function consists of the values a
random variable can assume and the corresponding probabilities of the values.

Example 1: Number of Heads

Suppose three coins are tossed. Let Y be the random variable representing the
number of heads that occur. Find the probability of each of the values of the random variable Y.

Solution: The sample space for this experiment is : S= {TTT,TTH,THT,HTT,HHT,HTH,THH,HHH}


H- Head T-Tail

Possible Outcomes TTT TTH THT HTT HHT HTH THH HHH
Values of the
0 1 1 1 2 2 2 3
Random Variable Y
There are four possible values of the random variable Y representing the
number of heads. These are 0,1,2, and 3.

What to Do Next? Assign probability values P(Y) to each value of the random variable and make a
table. You can write it in vertical/horizontal way.

5
Remember! Since there are 8 possible outcomes, and out of these, 1 outcome got no heads (TTT),
therefore the probability of getting 0 heads is 1/8. Do these also to other possible outcomes.

Number of Heads Y 0 1 2 3
1 3 3 1
Probability P(Y)
8 8 8 8
Table 1.1. The Probability Distribution or the Probability Mass Function of Discrete Random Variable Y

Example 2: Number of Blue Balls

Two balls are drawn in succession without replacement from an urn containing 5 red balls and
6 blue balls. Let Z be the random variable representing the number of blue balls. Construct the
probability distribution of the random variable Z.

Solution: The sample space for this experiment is: S = {RR, RB, BR, BB} , where R = Red, B = Blue
Value of Random Variable Z
Possible Outcomes
(Number of Blue Balls)
RR 0
RB 1
BR 1
BB 2
There are three possible values of the random variable Z representing the
number of blue balls. These are 0, 1, and 2.

Number of Blue Balls Z 0 1 2


1 2 1 1
Probability P(Z) or
4 4 2 4
Table 1.2. The Probability Distribution or the Probability Mass Function of Discrete
Random Variable Z

Can you make a histogram for this probability distribution?

Histogram for the Probability Distribution of Discrete Random Variable


Remember, a histogram is a bar graph. To construct a histogram for a probability distribution:

• Plot the values of the random variable (X, Y, etc.) along the horizontal axis.
• Plot the probabilities (P(X)) along the vertical axis.
Extra Tips:
1. If you are dealing with fractions, you can convert fractions to decimals for easy plotting.
2. In making the axes, make sure that you have equal intervals/spacing with the numbers.

Remember, ¼ is equivalent to 0.25 and ½ is equal to 0.50.

6
0.6

Probabilities P(Z)
0.5
0.4
0.3
0.2
0.1
0
0 1 2
Number of Blue Balls (Z)

Figure 1.1 The Histogram for the Probability Distribution of the Discrete Random Variable Z

Example 3. Number of Defective Cell Phones

Suppose three cell phones are tested at random. Let D represent the defective cell phone and
let N represent the non-defective cell phone. If we let X be the random variable for the number of
defective cell phones, construct the probability distribution of the random variable X.
Solution:
Possible Outcomes Value of the Random Variable X
(number of defective cell phones)
NNN 0
NND 1
NDN 1
DNN 1
NDD 2
DND 2
DDN 2
DDD 3

Table 1.3. Probability Distribution or Probability Mass function of Discrete Random Variable X.

Number of Defective Probability P(X)


Cell phones X
0 1
8
1 3
8
2 3
8
3 1
8

7
0.4
0.35

Probabilities P(X)
0.3
0.25
0.2
0.15
0.1
0.05
0
0 1 2 3
Number of Defective Cell Phones (X)

Figure 1.2. The Histogram for the Probability Distribution of the Discrete Random Variable Z

Properties of a Probability Distribution


1. The probability of each value of random variable must be between or equal to 0 and 1. In
symbol, 0 ≤ 𝑃(𝑋) ≤ 1 .
2. The sum of the probabilities of a value is equal to 1. In symbol, ∑ 𝑃(𝑋) = 1 .

Example 4: Daily Demand for Copies of a Movie Magazine

The daily demand for copies of a movie magazine at a variety store has the probability
distribution as follows:

Number of Copies X Probability P(X)


0 0.06
1 0.14
2 0.16
3 0.14
4 0.12
5 0.10
6 0.08
7 0.07
8 0.06
9 0.04
10 0.03

a. What is the probability that three or more copies will be demanded in a particular day?
b. What is the probability that the demand will be at least two but not more than six?
Solution:
a. 𝑃(𝑋 ≥ 3) = 0.14 + 0.12 + 0.10 + 0.08 + 0.07 + 0.06 + 0.04 + 0.03
𝑃(𝑋 ≥ 3) = 0.64

The probability that three or more copies will be demanded in a particular day is 0.64

b. 𝑃(2 ≤ 𝑋 ≤ 6) = 0.16 + 0.14 + 0.12 + 0.10 + 0.08


𝑃(2 ≤ 𝑋 ≤ 6) = 0.60
The probability that the demand will be at least two but not more than six is 0.60.

8
I. LEARNING COMPETENCIES
1. Illustrates a probability distribution for a discrete random variable and its properties.
M11/12SP-IIIa-4
2. Constructs the probability mass function of a discrete random variable and its corresponding
histogram. M11/12SP-IIIa-5
3. Computes probabilities corresponding to a given random variable. M11/12SP-IIIa-6

III. ACTIVITIES

Activity 1: YES or NO?

Directions: Determine whether the distribution represents a probability distribution. Write YES if it
is a probability distribution, otherwise write NO. Explain your answer.
1.
X 1 5 8 7 9
P(X) 1 1 1 1 1
3 3 3 3 3
2.
X 0 2 4 6 8
P(X) 1 1 1 1 1
6 6 3 6 6
3.
X 1 3 5 7
P(X) 0.35 0.25 0.22 0.12

4. P(1) = 0.42 , P(2) = 0.31, P(3) = 0.37


9 4 1
5. 𝑃(1) = ,P (2) = , 𝑃(3) =
14 14 14

Activity 2: CONSTRUCT ME!

Directions: Construct the probability distribution for the random variable described in each of the
following situations. Draw the corresponding histogram for each probability distribution.
Label it properly.
1. Let T be a random variable giving the number of tails in three tosses of a coin. List the elements
of the sample space S, assign a value to each sample point, and give its probability.
Elements of the Sample Value of the Random Variable T
Space S (Number of Tails)

Probability Distribution of the Discrete Random Variable T.


Number of Tails T Probability P(T)

9
2. The probabilities P(B) that a student will borrow 1, 2, 3, or 4 books are 0.45, 0.30,0.15, and 0.10,
respectively.

Activity 3: FIND ME!

The following data shows the probabilities for the number of cars sold in each day at a car
dealer store.

Number of Cars X Probability P(X)


0 0.100
1 0.150
2 0.250
3 0.140
4 0.090
5 0.080
6 0.060
7 0.050
8 0.040
9 0.025
10 0.015

1. Find 𝑃(𝑋 ≤ 2).


2. Find 𝑃(𝑋 ≥ 7).
3. Find 𝑃(1 ≤ 𝑋 ≤ 5).

Questions to Ponder
• How do you construct probability distribution?
_____________________________________________________________________
_____________________________________________________________________

• How do you make a histogram for a probability distribution? Give the steps in
constructing a histogram for probability distribution.
_____________________________________________________________________
_____________________________________________________________________

V. REFLECTION
1. What are your learnings from the activities?
___________________________________________________________________________

2. What did you like most from this lesson?


__________________________________________________________________________ .

3. Which of the activities did you find most challenging?


_________________________________________________________________________ .

VI. REFERENCES
Belecina, Rene R., et. al. (2016). Statistics and Probability First Edition, REX Book Store,
Inc., Florentino St., Sta. Mesa Heights, Quezon City, Philippines, pp. 9 – 20.

10
11
VII. ANSWER KEY
Activity 1: YES or NO?
1. NO, because the sum of the probabilities P(X) is 5/3 which not equal to 1.
2. YES, the sum of P(X) is equal to 1.
3. NO, because the sum of the probabilities P(X) is 0.94 which is not equal to 1.
4. NO, because the sum of the probabilities P(X) is 1.1 which is not equal to 1.
5. YES, the sum of P(X) is equal to 1.
Activity 2: CONSTRUCT ME!
1.
Value of the Random Variable T
Element of the sample space S
(Number of Tails)
HHH 0
HHT 1
HTH 1
THH 1
HTT 2
THT 2
TTH 2
TTT 3
Probability Distribution of the Discrete Random Variable T.
Number Tails
of T Probability P(T)
0 1
8
1 3
8
2 3
8
3 1
8
The Histogram for the Probability Distribution of the Discrete Random Variable T
12
Writer
MARIJO S. BALIN
Prepared by:
2.
Probability Distribution of the Discrete Random Variable B.
Value of the Random Variable B
Probability P(B)
(Number of Books)
1 0.45
2 0.30
3 0.15
4 0.10
The Histogram for the Probability Distribution of the Discrete Random Variable B
Activity 3: FIND ME
1. 𝑃(𝑋 ≤ 2) = 0.100 + 0.150 + 0.250
𝑷(𝑿 ≤ 𝟐) = 0.500
2. 𝑃(𝑋 ≥ 7) = 0.050 + 0.040 + 0.025 + 0.015
𝑷(𝑿 ≥ 𝟕) = 0.13
3. 𝑃(1 ≤ 𝑋 ≤ 5) = 0.150 + 0.250 + 0.140 + 0.090 + 0.080
𝑷(𝟏 ≤ 𝑿 ≤ 𝟓) = 0.71
STATISTICS AND PROBABILITY

Name of Learner: _____________________________ Section: _________________


Grade Level: _________________________________ Date: ___________________

Q3_W2_LAS3
COMPUTING THE MEAN OF A DISCRETE PROBABILITY DISTRIBUTION

I. BACKGROUND INFORMATION FOR LEARNERS

Hello dear young statistician! You are now on the second week of Quarter 3. In the
previous lesson you were able to construct probability distribution of discrete random variable
and a histogram.
This week, we will be exploring Mean of Probability Distribution. You have learned from
the previous study of mathematics how to find the mean of ungrouped and grouped data. You
also learned how to describe data using mean. In this lesson, you will learn how to compute the
mean of a discrete probability distribution. You will also learn how to interpret the mean of a
discrete probability distribution.

Now, it’s time for you to explore and learn. Good luck young statistician!
Mean of the Random Variable X or the Mean of the Probability Distribution of X
➢ The mean tells us the average number/ value that would appear.

Formula for the Mean of the Probability Distribution

The mean of a random variable with a discrete probability distribution is


𝜇 = 𝑋1 ∙ 𝑃(𝑋1 ) + 𝑋2 ∙ 𝑃(𝑋2 ) + ⋯ + 𝑋𝑛 ∙ 𝑃(𝑋𝑛 ) or
𝜇 = ∑ 𝑋 ∙ 𝑃(𝑋)
where: 𝑋1 , 𝑋2, 𝑋3,… 𝑋𝑛 are the values of the random variable X; and
𝑃(𝑋1 ), 𝑃(𝑋2), 𝑃(𝑋3)… 𝑃(𝑋𝑛 ) are the corresponding probabilities.

Example 1. Number of Spots


Consider rolling a die. What is the average number of spots that would appear?

Solution:
Steps Solution
1. Construct the probability distribution Number of Spots X Probability P(X)
for the random variable X representing 1 1
the number of spots that would appear. 6
2 1
6
3 1
6
4 1
6
5 1
6
6 1
6

13
2. Multiply the value of the random Number of Probability 𝑋 ∙ 𝑃(𝑋)
variable X by the corresponding Spots X P(X)
probability. 1 1 1
6 6
2 1 2
6 6
3 1 3
6 6
4 1 4
6 6
5 1 5
6 6
6 1 6
6 6
3. Add the results obtained in Step 2 21
∑ 𝑋 ∙ 𝑃 (𝑋 ) = = 3.5
6

The mean of the random variable X or the mean of the probability distribution of X is 3.5
This implies that the average number of spots that would appear in a roll of a die is 3.5.

Example 2. Grocery Items

The probabilities that a customer will buy 1, 2, 3, 4, or 5 items in a grocery


3 1 1 2 3
are 10
, , , , 𝑎𝑛𝑑 10, respectively. What is the average number of items that a customer will buy?
10 10 10

Solution:

Number of Items X Probability P(X) 𝑋 ∙ 𝑃(𝑋)


1 3 3
10 10
2 1 2
10 10
3 1 3
10 10
4 2 8
10 10
5 3 15
10 10
31
∑ 𝑋 ∙ 𝑃 (𝑋 ) = = 3.1
10

The mean of the probability distribution of X is 3.1.


This implies that the average number of items that the customer will buy is 3.1 or theoretically 3 items.

14
Example 3. Surgery Patients

The probabilities that a doctor operates on 3, 4, 5, 6, or 7 patients


in any day are 0.15, 0.10, 0.20, 0.25, and 0.30, respectively. Find the
average number of patients that a surgeon operates on a day.

Solution:
Number of Patients X Probability P(X) 𝑋 ∙ 𝑃(𝑋)
3 0.15 0.45
4 0.10 0.40
5 0.20 1.00
6 0.25 1.50
7 0.30 2.10
∑ 𝑋 ∙ 𝑃(𝑋) = 5.45
The average number of patients that a surgeon will operate in a day is 5.45.
This implies that the average number of patients that a surgeon will operate is 5.45 or
theoretically 5 patients.
II. LEARNING COMPETENCIES:
1. Illustrates the mean of a discrete random variable. M11/12SP-IIIb-1
2. Calculates the mean of a discrete random variable. M11/12SP-IIIb-2
3. Interprets the mean of a discrete random variable. M11/12SP-IIIb-3
4. Solves problem involving mean of a discrete probability distribution. M11/12SP-IIIb-4

III. ACTIVITIES

Activity 1: COMPLETE ME!

Complete the table below and find the mean of the probability distribution.
1.
X P(X) 𝑿 ∙ 𝑷(𝑿)
1 1
7
6 1
7
11 3
7
16 1
7
21 1
7
∑ 𝑋 ∙ 𝑃(𝑋) = ___________
The mean of the probability distribution is ______.
2.
X P(X) 𝑿 ∙ 𝑷(𝑿)
3 0.15
6 0.35
8 0.40
10 0.10
∑ 𝑋 ∙ 𝑃(𝑋) = ___________

The mean of the probability distribution is ______.

15
Activity 2: WHAT DO YOU MEAN?
Directions: Make a table for the situation below and find the mean of the probability distribution.
Show your complete solution.

The probabilities of a manufacturing 0, 1, 2, 3, 4, or 5 defective parts in one day are


0.75, 0.17, 0.04, 0.025, 0.01, and 0.005, respectively. Find the mean of the probability
distribution.

Activity 3: IS IT BELIEVABLE?
Directions: Read and analyze the situation below. Calculate the mean and probability distribution by
showing your complete solution and write your explanation.

A bakeshop owner determines the number of boxes of pandesal that are delivered
each day. Find the mean of the probability distribution shown. If the manager stated that 35
boxes of pandesal were delivered in one day, do you think that this a believable claim? Why?

Number of Boxes (X) Probability P(X)


35 0.10
36 0.20
37 0.30
38 0.30
39 0.10

Questions to Ponder
• What are the steps in computing the mean of a probability distribution of a random
variable? Compare these steps with the procedure in finding the mean of a frequency
distribution.
_____________________________________________________________________
_____________________________________________________________________
_____________________________________________________________________

∑ 𝑓𝑥
• Compare the formulas for finding the mean of a frequency distribution ( 𝑋 = 𝑚
)
𝑛
and that of a probability distribution ∑ 𝑋 ∙ 𝑃(𝑋).
_____________________________________________________________________
_____________________________________________________________________
_____________________________________________________________________
V. REFLECTION
Draw what you feel after doing the different activities on Computing the Mean of a Discrete
Probability Distribution and explain it.

VI. REFERENCES:
Belecina, Rene R., et. al. (2016). Statistics and Probability First Edition, REX Book Store, Inc.,
Florentino St., Sta. Mesa Heights, Quezon City, Philippines, pp. 21 – 30.

16
17
Writer
MARIJO S. BALIN
Prepared by:
VII. Answer Key
Activity #1 – COMPLETE ME!
1.
X P(X) 𝑋 ∙ 𝑃(𝑋)
1 1 𝟏
7 𝟕
6 1 𝟔
7 𝟕
11 3 𝟑𝟑
7 𝟕
16 1 𝟏𝟔
7 𝟕
21 1 𝟐𝟏
7 𝟕
𝟕𝟕
∑ 𝑿 ∙ 𝑷(𝑿) = or 11
𝟕
The mean of the probability distribution is 11.
2.
X P(X) 𝑋 ∙ 𝑃(𝑋)
3 0.15 0.45
6 0.35 2.1
8 0.40 3.2
10 0.10 1
∑ 𝑿 ∙ 𝑷(𝑿) = 6.75
The mean of the probability distribution is 6.75 .
Activity #2 – WHAT DO YOU MEAN?
No. of Defective Parts Probability 𝑋 ∙ 𝑃(𝑋)
X P(X)
0 0.75 0
1 0.17 0.17
2 0.04 0.08
3 0.025 0.075
4 0.01 0.04
5 0.005 0.025
∑ 𝑿 ∙ 𝑷(𝑿) = 0.39
The mean of the probability distribution is 0.39 .
Activity #3 – IS IT BELIEVABLE?
Number of Boxes (X) Probability P(X) 𝑋 ∙ 𝑃(𝑋)
35 0.10 3.5
36 0.20 7.2
37 0.30 11.1
38 0.30 11.4
39 0.10 3.9
∑ 𝑿 ∙ 𝑷(𝑿) = 37.1
The mean of the probability distribution is 37.1 .
No, It is unbelievable claim that the manager stated that 35 boxes of pandesal were
delivered in one day. It is actually 37.1 or 37 pandesal.
STATISTICS AND PROBABILITY

Name of Learner: _____________________________ Section: _________________


Grade Level: _________________________________ Date: ___________________

Q3_W3_LAS4
INTERPRETING THE VARIANCE OF A DISCRETE RANDOM VARIABLE

I. BACKGROUND INFORMATION FOR LEARNERS


You’ve learned from the previous Learning Activity Sheet how to find the Mean of a
Discrete Random Variable. This time, you will learn how to find the variance of a discrete random
variable, interpret the variance of a discrete random variable, and solve problems involving the
variance of probability distribution.
You have also learned that the mean of a random variable Χ is a measure of the central
location of the distribution of Χ. It is clear that location is not the only relevant feature. The second
most important feature is the spread of the distribution.
If values of Χ near its mean 𝜇 x are very likely and values further away from 𝜇 x have very
small probability, then the distribution of Χ will be closely concentrated around 𝜇 x. In this case,
the spread of the distribution of Χ is small. On the other hand, if values of Χ at some distance from
its mean 𝜇 x are likely, the spread of the distribution of Χ will be large. This idea leads to the most
important measure of spread, the variance and a closely related measure, the standard deviation.
For a start, here are the steps on how to find the Variance of a Discrete Random Variable. This will
serve as your guide as you do the activities later on.

Steps in Finding the Variance and Standard Deviation


1. Find the mean of the probability distribution.
2. Subtract the mean from each value of the random variable X.
3. Square the results obtained in Step 2.
4. Multiply the results obtained in Step 3 by the corresponding probability.
5. Get the sum of the results obtained in Step 4.

Formula for the Variance and Standard Deviation of a Discrete Probability


Distribution
The variance of a discrete probability distribution is given by the formula:
𝝈𝟐 = ∑(𝑿 − 𝝁)𝟐 ∙ 𝑷(𝑿)
The standard deviation of a discrete probability distribution is given by the formula:
𝝈 = √∑(X − μ)² ∙ P(X)
where:
X = value of the random variable
P(X) = probability of the random variable X
𝜇 = mean of the probability distribution

II. LEARNING COMPETENCIES:


1. Calculates the Variance of a Discrete Random Variable (M11/12SP-111b-2)
2. Interprets the Variance of a Discrete Random Variable (M11/12SP-111b-3)

18
3. Solves problems involving Variance of probability distribution (M11/12SP-IIIb-4)

Examples:

A. Number of Almonds
The random variable X, representing the number of almonds in a Fruit and Nut chocolate bar has the
following probability distribution. Compute the variance.

X 1 2 3 4 5

P (X) 2 3 3 1 1
10 10 10 10 10

Step 1: Find the mean of the probability distribution using the formula 𝜇 = ∑𝑋 ∙ (𝑋).

Number of Almonds Probability

X P (X) 𝑿 ∙ 𝑷(𝑿)
1 2 1•
2
=
𝟐
10 𝟏𝟎
10
2 3 2•
3
=
𝟔
10 𝟏𝟎
10
3 3 3•
3
=
𝟗
10 𝟏𝟎
10
4 1 4•
1
=
𝟒
10 𝟏𝟎
10
5 1 5•
1
=
𝟓
10 𝟏𝟎
10
𝟐𝟔
𝜇 = ∑𝑋 ∙ 𝑃(𝑋) = or 2.6
𝟏𝟎

Step 2. Subtract the mean from each value of the random variable X.

Number of Almonds Probability


X P(X) 𝑿 ∙ 𝑷(𝑿) 𝑿–𝜇
1 2 2 1 − 2.6 = −𝟏. 𝟔
10 10
2 3 6 2 − 2.6 = −𝟎. 𝟔
10 10
3 3 9 3 − 2.6 = 𝟎. 𝟒
10 10
4 1 4 4 − 2.6 = 𝟏. 𝟒
10 10
5 1 5 5 − 2.6 = 𝟐. 𝟒
10 10
26
𝜇 = ∑𝑋 ∙ 𝑃(𝑋) = 10 = 2.6

19
Step 3. Square the results obtained in Step 2.

Number of Almonds Probability 𝑿 ∙ 𝑷(𝑿) 𝑿–𝝁 (𝑿 – 𝝁)²


X P(X)
1 2 2 −1.6 (−1.6)² = 2.56
10 10
2 3 6 −0.6 (−0.6)² = 0.36
10 10
3 3 9 0.4 (0.4)² = 0.16
10 10
4 1 4 1.4 (1.4)² = 1.96
10 10
5 1 5 2.4 (2.4)² = 5.76
10 10
𝜇 = ∑𝑋 ∙ 𝑃(𝑋) =
2.6

Step 4. Multiply the results obtained in Step 3 by the corresponding probability.

Number of Probability
Almonds P(X)
X 𝑿 ∙ 𝑷(𝑿) 𝑿–𝜇 (𝑿 – 𝜇)² (𝑋 − 𝜇)² • 𝑷(𝑿)

1 2 2 −1.6 2.56 2
2.56 • = 0.512
10 10 10

2 3 6 −0.6 0.36 3
0.36 • = 0.108
10 10 10

3 3 9 0.4 0.16 3
0.16 • = 0.048
10 10 10

4 1 4 1.4 1.96 1
1.96 • = 0.196
10 10 10

5 1 5 2.4 5.76 1
5.76 • = 0.576
10 10 10

𝜇 = ∑𝑋 ∙ 𝑃(𝑋) = 2.6 𝜎2 = ∑(𝑋 − 𝜇)2 ∙ 𝑃(𝑋) = 1.44

Step 5. Get the square root of the variance to get the standard deviation.
The variance 𝜎2 of the probability distribution is 1.44
The standard deviation is 𝜎 =√𝑣𝑎𝑟𝑖𝑎𝑛𝑐𝑒 = 1.2

What does it mean?


The variance (1.44) is the weighted average of the squared deviation from the mean (2.6),
where the weights are given by the probability function.

20
B. Number of Face Masks

The number of boxes of face mask sold per day at a drugstore, with its corresponding
probabilities, is shown in the table. Find the variance and standard deviation of the probability
distribution.

Number of boxes sold Probability


X P (X)
19 0.20
20 0.20
21 0.30
22 0.20
23 0.10

Step 1: Find the mean of the probability distribution using the formula 𝜇 = ∑𝑋 ∙ (𝑋).

Number of Almonds Probability


X P (X) 𝑿 ∙ 𝑷(𝑿)
19 0.20 19•0.20 = 3.8
20 0.20 20•0.20 = 4.0
21 0.30 21•0.30 = 6.3
22 0.20 22•0.20 = 4.4
23 0.10 23•0.10 = 2.3
𝜇 = ∑𝑋 ∙ 𝑃(𝑋) = 20.8

Step 2. Subtract the mean from each value of the random variable X.

Number of Almonds Probability


X P (X) 𝑿 ∙ 𝑷(𝑿) 𝑿−𝜇
19 0.20 3.8 19 – 20.8 = -1.8
20 0.20 4.0 20 – 20.8 = -0.8
21 0.30 6.3 21 – 20.8 = 0.2
22 0.20 4.4 22 – 20.8 = 1.2
23 0.10 2.3 23 – 20.8 = 2.2
𝜇 = ∑𝑋 ∙ 𝑃(𝑋) = 20.8

Step 3. Square the results obtained in Step 2.

Number of Probability
Almonds P (X) 𝑿 ∙ 𝑷(𝑿) 𝑿−𝜇 (𝑿 – 𝜇)²
X
19 0.20 3.8 -1.8 (-1.8)² = 3.24
20 0.20 4.0 -0.8 (-0.8)² = 0.64
21 0.30 6.3 0.2 (0.2)² = 0.04
22 0.20 4.4 1.2 (1.2)² = 1.44
23 0.10 2.3 2.2 (2.2)² = 4.84
𝜇 = ∑𝑋 ∙ 𝑃(𝑋) = 20.8

Step 4. Multiply the results obtained in Step 3 by the corresponding probability.

21
Number of Probability
Almonds P (X) 𝑿 ∙ 𝑷(𝑿) 𝑿−𝜇 (𝑿 – 𝜇)² (𝑋 − 𝜇)² • 𝑷(𝑿)
X
19 0.20 3.8 -1.8 3.24 3.24 • 0.20 = 0.648
20 0.20 4.0 -0.8 0.64 0.64 • 0.20 = 0.128
21 0.30 6.3 0.2 0.04 0.04•0.30 = 0.012
22 0.20 4.4 1.2 1.44 1.44 • 0.20 = 0.288
23 0.10 2.3 2.2 4.84 4.84 • 0.10 = 0.484
𝜇 =∑𝑋 ∙ 𝑃(𝑋) ∑(𝑿 − 𝝁)𝟐 ∙ 𝑷(𝑿) = 1.56
= 20.8

Step 5. Get the square root of the variance to get the standard deviation.
The variance of the probability distribution is 1.56.
The standard deviation is 𝜎 = √1.56 = 1.25.

III. ACTIVITIES
Activity 1. Calculate Me!
1. Number of Heads
When three coins are tossed, the probability distribution for the random variable X
representing the number of heads that occur is given below. Compute the variance and standard
deviation of the probability distribution.

Number of Probability
Heads P(X) 𝑿 ∙ 𝑷(𝑿) 𝑿−𝜇 (𝑿 – 𝜇)² (𝑋 − 𝜇)² • 𝑷(𝑿)
X
0 1
8
1 3
8
2 1
8
3 3
8
𝜇 =∑𝑋 ∙ 𝑃(𝑋) = _____ 𝜎² = ∑(𝑿 − 𝝁)𝟐 ∙ 𝑷(𝑿) = __________

The variance of the probability distribution is _____.


The standard deviation is _______.

2. Number of Cakes
A Bake shop has the following schedule of daily demand for cakes. Find the expected
number of cakes demands per day.
Number of cakes demands (in hundred) Probability
X P (X)
0 0.10
1 0.15
2 0.20
3 0.25
4 0.20
5 0.10

22
Number of
Probability
cakes 𝑿−𝜇
P(X) 𝑿 ∙ 𝑷(𝑿) (𝑿 – 𝜇)² (𝑋 − 𝜇)² • 𝑷(𝑿)
demands (X)
0 0.10
1 0.15
2 0.20
3 0.25
4 0.20
5 0.10
𝜇 = ∑𝑋 ∙ 𝑃(𝑋) = ___ 𝜎² = ∑(𝑿 − 𝝁)𝟐 ∙ 𝑷(𝑿) = __________

The variance of the probability distribution is _____.


The standard deviation is ______.

I. What I Have Learned


Answer the following questions. Write your answers on the space provided.
1. How do you interpret the variance of a probability distribution?
__________________________________________________________________________________
_________________________________________________________________________________.
2. How do you solve problems involving variance of probability distribution? Give the step-by-step procedure.
__________________________________________________________________________________
_________________________________________________________________________________.

IV. REFLECTION

1. What have you learned from this activity?


______________________________________________________________________________________
______________________________________________________________________________________

2. Have you had difficulty in dealing with answering the activities?


______________________________________________________________________________________
______________________________________________________________________________________

3. What do you like the most with this activity sheet?


______________________________________________________________________________________
______________________________________________________________________________________

VI. REFERENCES

Belecina, Rene R. et. al. Statistics and Probability. P. Florentino ST., Sta. Mesa Heights, Quezon City:
Rex Printing Company, Inc., 2016

Retrieved from https://amsi.org.au/ESA_Senior_Years/SeniorTopic4/4c/4c_2content_6.

23
24
Writer
AMIE-LYN S. CEDO
Prepared by:
1.
X P(X) 𝑿 ∙ 𝑷(𝑿) 𝑿−𝜇 (𝑿 – 𝜇)² (𝑋 − 𝜇)² • 𝑷(𝑿)
0 1 0 -1.75 3.06 0.383
8
1 3 3 -0.75 0.56 0.21
8 8
2 1 2 0.25 0.06 0.0075
8 8
3 3 9 1.25 1.56 0.585
8 8
14 𝜎² = ∑(𝑿 − 𝝁)𝟐 ∙ 𝑷(𝑿) = 1.1855 = 1.19
𝜇 = ∑𝑋 ∙ 𝑃(𝑋) = =
8
1.75
The variance = 1.19
The standard deviation = √1.19 = 1.09
2.
X P(X) 𝑿 ∙ 𝑷(𝑿) 𝑿−𝜇 (𝑿 – 𝜇)² (𝑋 − 𝜇)² • 𝑷(𝑿)
0 0.10 0 -2.6 6.76 0.676
1 0.15 0.15 -1.6 2.56 0.384
2 0.20 0.4 -0.6 0.36 0.072
3 0.25 0.75 0.4 0.16 0.040
4 0.20 0.80 1.4 1.96 0.392
5 0.10 0.50 2.4 5.76 0.576
𝜇 = ∑𝑋 ∙ 𝑃(𝑋) = 𝜎² = ∑(𝑿 − 𝝁)𝟐 ∙ 𝑷(𝑿) = 2.14
2.6
The variance = 2.14
The standard deviation = √2.14 = 1.46
STATISTICS AND PROBABILITY

Name of Learner: _____________________________ Section: _________________


Grade Level: _________________________________ Date: ___________________

Q3_W3_LAS5
ILLUSTRATING NORMAL DISTRIBUTION

I. BACKGROUND INFORMATION FOR LEARNERS


You’ve learned from the previous Learning Activity Sheet the concept of random variables that
focuses on discrete probability distribution. The probability of a distribution enables to make
predictions and decisions concerning the populations. However, there are data that are continuous.
The distribution of this type is known as the normal probability distribution or the normal distribution.
Data can be distributed in a variety of ways. It can spread more on the left or more on the right
or even jumbled up. But there are many cases where the data tends to be around a central value
with no bias left or right, and it gets close to a "Normal Distribution". L o o k a t the model shown
below.

Fig. 1. The Normal Probability Distribution


The given distribution consists of a large number of cases and the three measures of
central tendency (mean, median, and mode) are equal and the distribution is symmetrical and the
skewness is 0. In Statistics, such distribution is called normal distribution or simply normal curve.The
normal distribution is the most important and most widely used distribution in statistics. It is the most
common type of data distribution and is also known as bell curve. Most observations are centered
around the mean and fewer data observation beyond either side of the mean.

Properties of the Normal Probability Distribution

The normal probability distribution has the following properties


1. The distribution curve is bell-shaped.
2. The curve is symmetrical
about its center. The mean, the
median, and the mode are equal and
coincide at the center.
3. The width of the curve is determined by the standard
deviation of the distribution.
4. The tails of the curve flatten out indefinitely along the horizontal
axis, always approaching the axis but never touching it. That is, the
curve is asymptotic to the base line.

25
5. The area under the curve is 1. Exactly half of the values are to the left
of the center and exactly half the values are to the right.
6. Empirical rule, it has a distribution with 68% of observation in the population
lie within 𝜇 ± 𝜎, 95% of observations in the population lie within 𝜇 ± 2𝜎 and
99.7% of observations in the population lie within 𝜇 ± 3𝜎.

The Normal Random Variable


The equation that describes normal curve is:

1 1 𝓍−𝜇 2
( 𝜎 )
𝑓(𝑥) = ℯ2 , −∞ < 𝓍 < +∞
𝜎√2𝜋

where:
µ = constant, , −∞ < 𝜇 < +∞ (mean of X)
𝜎 = constant, 𝜎 > 0 (standard deviation of X)
𝜋 = 3.1416
ℯ = 2.7183

The Standard Normal Distribution


A normal distribution is determined by the mean 𝜇 and standard deviation 𝜎. If the mean µ = 0
and a standard deviation σ = 1, the normal distribution is called a standard normal distribution. By
substituting the mean, µ = 0 and the standard deviation, σ = 1 in the formula mathematicians are able
to find the areas under the normal curve. The areas under the normal curve can be found using the
Areas under the Standard Normal Distribution Table. These areas have already been predetermined
for use.

Mean
Fig. 2.2. The Areas Under the Normal Curve
The middle regions under the normal curve are shown above. The total area between z = −1
and z =1 is 2(0.3413) = 0.6826 or 68.26%; between 𝑧 = −2 and 𝑧 = 2, the total area is 0.9544 or 95.44%;
and the total area between 𝑧 = −3 and 𝑧 = 3 is 0.9974 or 99.74%. This means that we can determine
the area in any specified region under the normal curve and associate it with probability, proportion,
and percentage.

Using a Standard Normal Distribution Table

The table of areas under the normal curve is also known as the z-Table. The z-score is a
measure of relative standing. It is calculated by subtracting 𝑥 or 𝜇 from the measurement of 𝑥 and
then dividing the result by 𝑠 or σ. The final result, the z-score, represents the distance between a given

26
measurement 𝑥 and the mean, expressed in standard deviations. Either z-score locates 𝑥 within a
sample within a population.

z .00 .01 .02 .03 .04 .05 .06 .07 .08 .09
0.0 0.0000 0.0040 0.0080 0.0120 0.0160 0.0199 0.0239 0.0279 0.0319 0.0359
0.1 0.0398 0.0438 0.0478 0.0517 0.0557 0.0596 0.0636 0.0675 0.0714 0.0753
0.2 0.0793 0.0832 0.0871 0.0910 0.0948 0.0987 0.1026 0.1064 0.1103 0.1141
0.3 0.1179 0.1217 0.1255 0.1293 0.1331 0.1368 0.1406 0.1443 0.1480 0.1517
0.4 0.1554 0.1591 0.1628 0.1664 0.1700 0.1736 0.1772 0.1808 0.1844 0.1879
0.5 0.1915 0.1950 0.1985 0.2019 0.2054 0.2088 0.2123 0.2157 0.2190 0.2224
0.6 0.2257 0.2291 0.2324 0.2357 0.2389 0.2422 0.2454 0.2486 0.2517 0.2549
0.7 0.2580 0.2611 0.2642 0.2673 0.2704 0.2734 0.2764 0.2794 0.2823 0.2852
0.8 0.2881 0.2910 0.2939 0.2967 0.2995 0.3023 0.3051 0.3078 0.3106 0.3133
0.9 0.3159 0.3186 0.3212 0.3238 0.3264 0.3289 0.3315 0.3340 0.3365 0.3389
1.0 0.3413 0.3438 0.3461 0.3485 0.3508 0.3531 0.3554 0.3577 0.3599 0.3621
1.1 0.3643 0.3665 0.3686 0.3708 0.3729 0.3749 0.3770 0.3790 0.3810 0.3830
1.2 0.3849 0.3869 0.3888 0.3907 0.3925 0.3944 0.3962 0.3980 0.3997 0.4015
1.3 0.4032 0.4049 0.4066 0.4082 0.4099 0.4115 0.4131 0.4147 0.4162 0.4177
1.4 0.4192 0.4207 0.4222 0.4236 0.4251 0.4265 0.4279 0.4292 0.4306 0.4319
1.5 0.4332 0.4345 0.4357 0.4370 0.4382 0.4394 0.4406 0.4418 0.4429 0.4441
1.6 0.4452 0.4463 0.4474 0.4484 0.4495 0.4505 0.4515 0.4525 0.4535 0.4545
1.7 0.4554 0.4564 0.4573 0.4582 0.4591 0.4599 0.4608 0.4616 0.4625 0.4633
1.8 0.4641 0.4649 0.4656 0.4664 0.4671 0.4678 0.4686 0.4693 0.4699 0.4706
1.9 0.4713 0.4719 0.4726 0.4732 0.4738 0.4744 0.4750 0.4756 0.4761 0.4767
2.0 0.4772 0.4778 0.4783 0.4788 0.4793 0.4798 0.4803 0.4808 0.4812 0.4817
2.1 0.4821 0.4826 0.4830 0.4834 0.4838 0.4842 0.4846 0.4850 0.4854 0.4857
2.2 0.4861 0.4864 0.4868 0.4871 0.4875 0.4878 0.4881 0.4884 0.4887 0.4890
2.3 0.4893 0.4896 0.4898 0.4901 0.4904 0.4906 0.4909 0.4911 0.4913 0.4916
2.4 0.4818 0.4920 0.4922 0.4925 0.4927 0.4929 0.4931 0.4932 0.4934 0.4936
2.5 0.4938 0.4940 0.4941 0.4943 0.4945 0.4946 0.4948 0.4949 0.4951 0.4952
2.6 0.4953 0.4955 0.4956 0.4957 0.4959 0.4960 0.4961 0.4962 0.4963 0.4964
2.7 0.4965 0.4966 0.4967 0.4968 0.4969 0.4970 0.4971 0.4972 0.4973 0.4974
2.8 0.4974 0.4975 0.4976 0.4977 0.4977 0.4978 0.4979 0.4979 0.4980 0.4981
2.9 0.4981 0.4982 0.4982 0.4983 0.4984 0.4984 0.4985 0.4985 0.4986 0.4986
3.0 0.4987 0.4987 0.4987 0.4988 0.4988 0.4989 0.4989 0.4989 0.4990 0.4990
3.1 0.4990 0.4991 0.4991 0.4991 0.4992 0.4992 0.4992 0.4992 0.4993 0.4993
3.2 0.4993 0.4993 0.4994 0.4994 0.4994 0.4994 0.4994 0.4995 0.4995 0.4995
3.3 0.4995 0.4995 0.4995 0.4995 0.4996 0.4996 0.4996 0.4996 0.4996 0.4997
3.4 0.4997 0.4997 0.4997 0.4997 0.4997 0.4997 0.4997 0.4997 0.4997 0.4998
3.5 0.4998 0.4998 0.4998 0.4998 0.4998 0.4998 0.4998 0.4998 0.4998 0.4998

Example 1:

Elena conducted a survey and gathered data regarding daily travel time in minutes. After
assuming the data is normally distributed, she found out that the daily travel time has a mean of 32.28
minutes with a standard deviation of 10.68 minutes.

µ = mean daily travel time in minutes standard deviation of daily time travel in minutes
𝜎 = standard deviation of daily time travel in minutes
X = daily time travel in minutes
The data is assumed to be normally distributed, hence, it has a symmestric property. Since mean
µ = 32.28, therefore the mode = median = 32. 28

27
Example 2:

Teacher Jared conducted a 100-item pre-test to his 30 students. He then marks the tests and
gathered the scores of his students. Assuming that the population is normally distributed, 95% of the
students got a score between 70 point and 89 points.

A is a normal continuous random variable which is equal to the exam scores. The variable A
may take on any value within the range of (70,89). Some may have exam scores that is exactly
70 or 89 or even other integer value within that range. However, some may have scores that
have a decimal value that is still within the said range. There are infinitely many possible values of A
that’s within the set range.

Computing for the mean:


The mean is halfway between 70 and 89
70 + 89
𝜇= = 79.5
2
Therefore, the mean exam scores of the students are equal to 79.5
Computing for the standard deviation:

95% is 2 standard deviations (2𝜎) either side of mean, meaning a total of 4 standard deviation.
89 − 70
𝜎= = 4.75
4
Therefore, the standard deviation of exams of the students is equal to 4.75.
Example 3:

The length (in days) of a randomly chosen human pregnancy is a normal random variable with
𝜇 = 266, 𝜎= 16. So, X = length of pregnancy in days. What is the probability that a randomly chosen
pregnancy will last less than 246 days?

We want 𝑃(𝑋 < 246). To find this probability, we first convert 𝑋= 246 to a Z-score.
246−266 −20
𝑍= = =
16 16
−1.25

This is the area under the normal


probability curve
to the left of 𝑍 = −1.25

Thus, the probability that a randomly chosen pregnancy will last less than 246 days is 0.1056. In
other words, there is an 11% chance that a randomly selected pregnancy will last less than 246 days.

28
II. LEARNING COMPETENCIES
1. Defines normal random variable
2 Identifies the normal probability distribution function.
3. Illustrates normal random variable and its characteristics (M11/12SP-IIIc-1)

III. ACTIVITIES

A. Many things closely follow a normal distribution. Give five examples of real-life data which follow this
type of pattern. Example: heights of people
1. _____________________________________________
2. ______________________________________________
3. ______________________________________________
4. ______________________________________________
5. ______________________________________________

B. Encircle the correct answer.

1. Children’s heights are normal with 𝜇 = 36 inches and 𝜎 = 2 inches. So, a child 39.5 inches tall has a
standardized height of ____.
A. 1.75 B. 18 C. 1.10 D. none of the choices
2. In February 2021, 1,664,479 persons took the COVID vaccine in Asia. The distribution of Pfizer
vaccine had a 𝜇 = 496 and a 𝜎 = 144. Find 𝑍 if X = 366.
A. 1.14 B. 4.35 C. 2.54 D. none of the choices
3. Now that there is a pandemic, some doctors believe that a person can be healthy if he or she can lose
five pounds, on the average, in a month by reducing his or her fat intake and by exercising regularly.
Suppose a person lost ten pounds in a month. Use a 𝜎 = 2. What is 𝑍?
A. 2.5 B. -2.5 C. 5 D. none of the choice

C. Fill in the blanks with the appropriate word or phrase to make a meaningful statement.
1. The normal distribution curve is a ___________ -shaped probability distribution.
2. The area under the normal curve is ___________.
3. The width of the curve is determined by the ___________ of the distribution.
4. The curve is ___________ to the base line.
5. The curve is ___________ about its center.
6. The mean, median, and mode of the normal curve are ___________.
7-8. The standard normal curve is a normal probability distribution that has a mean of
(7) _______________ and standard deviation of (8) ____________.

GUIDE QUESTIONS:

1. What did you understand about normal distribution?


___________________________________________________________________________
2. What are the characteristics of a normal distribution?
___________________________________________________________________________________
______________________________________________________.

29
IV. REFLECTION:

1. What have you learned from this activity?


___________________________________________________________________________________
_________________________________________________________________.

2. Have you had difficulty in dealing with answering the activities?


___________________________________________________________________________________
________________________________________________________________.

3. What do you like the most with this activity sheet?


___________________________________________________________________________________
__________________________________________________________________.

V. REFERENCES

Belecina, Rene, E. Baccay, and E. Mateo. Statistics and Probability. Manila: Rex Book Store, Inc., 2016.

Glen, Stephanie. "Normal Distributions (Bell Curve): Definition, Word Problems".


StatisticsHowTo.com: Elementary Statistics for the rest of us! Accessed May 22, 2020.
https://www.statisticshowto.com/probability-and-statistics/normal distributions/

https://prezi.com/r3kegqwojjpd/illustrating-a-normal-random-variable-and-its-characteristics/

https://prezi.com/xsr8u2zrs3dl/normal-distribution/

https://courses.lumenlearning.com/wmopen-concepts-statistics/chapter/introduction-to-normal-
random-vairables-6-of-6/

https://courses.lumenlearning.com/introstats1/chapter/the-standard-normal-distribution/

Answer Key:

8. 1
7. 0
6. Equal
5. Symmetric
4. Asymptotic
3. Standard deviation
2. 1
C. 1. bell
3. A
2. D
B. 1. A
A. Learner’s answer may vary

Prepared by:

AMIE-LYN S. CEDO
Writer

30
STATISTICS AND PROBABILITY

Name of Learner: _____________________________ Section: _________________


Grade Level: _________________________________ Date: ___________________

Q3_W4_LAS6
IDENTIFYING AREAS UNDER THE NORMAL CURVE

I. BACKGROUND INFORMATION FOR LEARNERS


In the previous lesson, you have learned about the properties of the normal
distribution. The distribution curve is bell-shaped, symmetrical about its center. The mean,
the median, and the mode are equal and coincide at the center, the width of the curve is
determined by the standard deviation of the distribution, the curve is asymptotic to the
base line, the area under the curve s 1 and it represents the probability or proportion
or the percentage associated with the specific sets of measurement values.

You have also learned how to read entries in the z-table and how to use it in
determining the area under the normal curve. This time, you will enrich your learning on
identifying regions under the normal curve corresponding to different standard normal
values. When you say region under the curve, you are interested in the area of the region.

The Standard Normal Distribution normally distributed variable has its own mean and
standard deviation, as stated earlier, the shape and location of these curves will vary. In
practical applications, then, you would have to have a table of areas under the curve for each
variable. To simplify this situation, statisticians use what is called the standard normal
distribution.
The curve shows a shaded region between 𝑧 = 0 and
𝑧 = 1. Similarly, a region between 𝑧 = 0 and 𝑧 = −1 has
the same area.

The shaded region between z= 0 and z= 2 is the same


with the region between z=0 and z=-2

Regions under the curve can be described in terms of area. Area between two
specific z-values can be determined using the z-table and the suggested steps below.

Steps in Identifying Region under a Normal Curve:


1.Draw the normal curve and locate the given z-value or values at the base line of the curve.
Then, draw a vertical line through the given z-value or values and shade the required region.
2. Use the z-table to find the areas that correspond to the given z-value or values.
3. Perform appropriate operations to get the required area, if needed.
4. Write the required area.

Case 1. When the required area is between 𝑧 = 0 and any z-value:

31
Example 1: Find the area that corresponds to 𝑧 = −1 is the same as finding the area between 𝑧 = 0
and 𝑧 = −1.

1. Draw the normal curve and locate the given z-


value or values at the base line of the curve.
Then, draw a vertical line through the given z-
value or values and shade the required region.

2. Use the z-table to find the areas that 𝑧 = −1 corresponds to an area of 0.3413
correspond to the given z-value or values.

3. Perform appropriate operations to get the No operations needed because the answer
required area, if needed. is already given in the z-table.

4. Write the required area. Thus, the area that corresponds to 𝑧 = −1


is 0.3413.

Example 2: Find the area that corresponds to 𝑧 = 1.

1. Draw the normal curve and locate the given z-


value or values at the base line of the curve.
Then, draw a vertical line through the given z
value or values and shade the required region.
2. Use the z-table to find the areas that 𝑧 = 1 corresponds to an area of
correspond to the given z-value or values. 0.3413
3. Perform appropriate operations to get the No operations needed because the
required area, if needed. answer is already given in the z-table.
4. Write the required area. Thus, the area that corresponds to 𝑧 = 1
is 0.3413.

Remember: When z is negative, simply ignore the sign. The negative informs us that the region is
found on the left side of the mean. Remember, areas are positive values.

Case 2. When the required area is greater than z

Other than the terms “greater than”, you can use these words to convey the same meaning: at least
z, more than z, to the right of z, or above z.

32
Example 3: Find the area above𝑧 = −1.34.

1. Draw the normal curve and locate the given z-


value or values at the base line of the curve.
Then, draw a vertical line through the given z
value or values and shade the required region.

2. Use the z-table to find the areas that 𝑧 = −1.34 corresponds to an area of 0.4099
correspond to the given z-value or values.
3. Perform appropriate operations to get the The graph suggests addition.
required area, if needed. 0.4099 + 0.5 = 0.9099
4. Write the required area. Thus, the area above 𝑧 = −1.34 is 0.9099.

Example 4: Find the area to the right of 𝑧 = 1.56

1. Draw the normal curve and locate the given z-


value or values at the base line of the curve.
Then, draw a vertical line through the given z
value or values and shade the required region.

2. Use the z-table to find the areas that 1.56 corresponds to an area of 0.4406.
correspond to the given z-value or values.
3. Perform appropriate operations to get the The graph suggests subtraction.
required area, if needed. 0.5 − 0.4406 = 0.0594
4. Write the required area. Thus, the area to the right of 𝑧 = 1.56 is
0.4406.

Case 3. When the required area is less than z

Other than the terms “less than”, you can use these words to convey the same meaning: at most z, no
more than z, not greater than z, or to the left of z

Example 5: Find the area to the left of 𝑧 = −1.52


1. Draw the normal curve and locate the given z-
value or values at the base line of the curve.
Then, draw a vertical line through the given z-
value or values and shade the required region.

2. Use the z-table to find the areas that 𝑧 = −1.52 corresponds to an area of 0.4357.
correspond to the given z-value or values.
3. Perform appropriate operations to get the The graph suggests subtraction. 0.5 − 0.4357
required area, if needed. = 0.0643
4. Write the required area. Thus, the area to the left of 𝑧 = −1.52 is
0.0643.

33
Example 6: Find the area less than 𝑧 = 1.25

1.Draw the normal curve and locate the given z-


value or values at the base line of the curve.
Then, draw a vertical line through the given z-
value or values and shade the required region.

2. Use the z-table to find the areas that 𝑧 = 1.25 corresponds to an area of 0.3944
correspond to the given z-value or values.
3. Perform appropriate operations to get the The graph suggests addition. 0.5 + 0.3944 =
required area, if needed. 0.8944
4. Write the required area. Thus, the area less than 𝑧 = 1.25 is 0.8944.

Case 4. When the required area is between two z-values of the same sign.

Example 7: Find the area between 𝑧 = 1.70 and 𝑧 = 0.82.

1. Draw the normal curve and locate the given z-


value or values at the base line of the curve.
Then, draw a vertical line through the given z
value or values and shade the required region.

2. Use the z-table to find the areas that 𝑧 = 1.70 corresponds to an area of 0.4554
correspond to the given z-value or values. 𝑧 = 0.82 corresponds to an area of 0.2939
3. Perform appropriate operations to get the The graph suggests subtraction. Subtract the
required area, if needed. smaller area from the larger area. 0.4554 −
0.2939 = 0.1615
4. Write the required area. Thus, the area between 𝑧 = 0.82 and 𝑧 = 1.70
is 0.1615.

Example 8: Find the area between 𝑧 = −2 and 𝑧 = −1

1. Draw the normal curve and locate the given z-


value or values at the base line of the curve.
Then, draw a vertical line through the given z-
value or values and shade the required region.
2. Use the z-table to find the areas that 𝑧 = −2 corresponds to an area of 0.4772 𝑧 = −1
correspond to the given z-value or values corresponds to an area of 0.3413
3. Perform appropriate operations to get the The graph suggests subtraction. Subtract the
required area, if needed. smaller area from the larger area. 0.4772 −
0.3413 = 0.1359
4. Write the required area. Thus, the area between 𝑧 = −2 and 𝑧 = −1 is
0.1359.

34
Case 5. When the required area is between −z 1 and z2

Example 9: Find the area between 𝑧 = −1.54 and z = 1.75

1. Draw the normal curve and locate the given z-


value or values at the base line of the curve.
Then, draw a vertical line through the given z
value or values and shade the required region.

2. Use the z-table to find the areas that 𝑧 = −1.54 corresponds to an area of 0.4382 𝑧
correspond to the given z-value or values. = 1.75 corresponds to an area of 0.4599
3. Perform appropriate operations to get the The graph suggests addition. 0.4382 + 0.4599
required area, if needed. = 0.8981
4. Write the required area Thus, the area between 𝑧 = −1.54 and 𝑧 = 1.75
is 0.8981.

Let’s extend your learning!


Now, let us extend your learning on representing the region under the normal curve in terms
of percent. Sketch the normal curve and use the z-table to find the area that corresponds to each of the
following z-values. Then, multiply the area to 100 to get its percent form. The final answers are given as
your guide.
1. Find the area between z= 1.68 and z =-1.37.

1. Draw the normal curve and locate the given z-


value or values at the base line of the curve.
Then, draw a vertical line through the given z-
value or values and shade the required region.
2. Use the z-table to find the areas that z= 1.68 corresponds to an area of 0.4535
correspond to the given z-value or values. z= -1.37 corresponds to an area of 0.1443
3. Perform appropriate operations to get the The graph suggests Addition.
required area, if needed. To convert area to 0.4535+ 0.1443= 0.8682
percent, multiply the area to 100.
4. Write the required area (in percent). Thus, the area between 𝑧 = 1.68 and 𝑧 = -
1.37 is 86.82%
2. Find the area to the left of z= 2.06.
1. Draw the normal curve and locate the given z-
value or values at the base line of the curve.
Then, draw a vertical line through the given z-
value or values and shade the required region.
2. Use the z-table to find the areas that z= 2.06 corresponds an area of 0.4803
correspond to the given z-value or values.

35
3. Perform appropriate operations to get the The graph suggest addition.
required area, if needed. To convert area to 0.5 + 0.4803= 0.9803
percent, multiply the area to 100.
4. Write the required area (in percent). Thus, the area to the left of z=2.06 is
98.03%

Let’s summarize!

• When speaking about a region under the curve, we are interested in the area of the region.
• The area in any specified region under the normal curve can be associated with
probability, proportion, or percentage.
• When 𝑧 is negative, simply ignore the negative sign and proceed. The negative sign indicates
that the region is on the left side of the mean or below the mean. Areas are always positive value.

I. LEARNING COMPETENCIES:
1. Identifies region under the normal curve corresponding to different standard normal vales.
M11/12SP-111C3
2. Find and interpret the area under a normal curve.

II. ACTIVITIES
ACTIVITY 1: WHAT I CAN DO
Answer the questions below.

1. What is the total area under the standard normal distribution curve?
_________________________________________________________________
2. What percentage of the area falls below the mean? Above the mean?
_________________________________________________________________
_______________________________________________________________________

3. About what percentage of the area under the normal distribution curve that falls within 1
standard deviation above the mean? 2 standard deviations below the mean? 3 standard
deviations above the mean?
_____________________________________________________________________________
_____________________________________________________________________________

ACTIVITY 2: FIND ME!


Answer the following questions based on the data.

2000 Grade 11 students of MORMS took a Statistics test. The scores were distributed normally
with a mean of 70 and Standard deviation of 5. Label the mean and three standard deviations from the
mean.

36
1.What percentage of scores are between 65 and 75?
2.What percentage of scores are between 60 and 70?
3.What percentage of scores less than a score of 60?
4.What percentage of scores greater than a score of
80?
5. What percentage of score are between 70 and 75?

ACTIVITY 3: DRAW ME!


Directions: Draw the normal curve, then locate the given z values in the curve. Find the area that
corresponds to the given z values using the z-table, then shade the required area.

1. Between z= 0 and z= 1.77


2. To the right of z = 2.01
3. To the left of z=- 0.75
4. Between z= 0 and z= 0.75
5. To the left of z =1.39

IV. REFLECTION

1. What have you learned from this activity?


_____________________________________________________________________
2. What did you like most from this lesson?
_____________________________________________________________________
3. What is the most challenging or least interesting from the activities?
____________________________________________________________________________

V. REFERENCES:

Belecina, Rene, E. Baccay, and E. Mateo (2016). Statistics and Probability: Rex Book Store, Inc., Manila
Philippines.
Glen, Stephanie "Normal Distributions (Bell Curve): Definition, Word Problems".
StatisticsHowTo.com: Elementary Statistics for the rest of us!Retrived from URL on May 22,.
Bluman, Allan G. (2012) Elementary Statistics:A Step by Step Approach, Eighth Edition, McGraw-Hill
Companies Inc., 1221 Avenue of the Americas, New York, pp.345-347

37
38
Writer
CHARMIE G. RELLORES
Prepared by:
Answer Key:
Activity 1: What I Can Do?
1. The total area under the standard normal curve is 1.00 or 100%
2. 50% above the mean, 5o % below the mean.
3. 1 standard deviation above the mean has an area of 34.13%.
2 standard deviations below the mean with an area of 47.72%.
3 standard deviations above the mean with an area of 50.00%
Activity 2: Find Me!
55 60 65 70 75 80 85
1. Between 65 & 75? 68.26%
2. Between 60 & 70? 47.72%
3. Less than 60? 2.28%
4. Greater than 80? 2.28%
5. Between 70 and 75? 34.13%
Activity 3: Draw Me!
1. Between z = 0 and z = 1.77 46.16%
2. To the right of z = 2.01 2.22%
3. To the left of z = -0.75 22.66%
4. Between z = 0 and z = 0.75 27.34%
5. To the left of z = 1.39 91.77%
STATISTICS AND PROBABILITY

Name of Learner: _____________________________ Section: _________________


Grade Level: _________________________________ Date: ___________________

Q3_W4_LAS7
CONVERTING NORMAL RANDOM VARIABLE TO A STANDARD NORMAL VARIABLE
AND VICE VERSA

I. BACKGROUND INFORMATION FOR LEARNERS


In the previous lessons, you have learned about the basic concept of normal distribution.
You further learned on how to identify the regions under the normal curve using the z-table.
In this lesson, you will learn how to relate the concept of the normal curve to a random variable
distribution using the z-score. You will be transforming a normal random variable to a standard
normal variable and vice-versa.

What do you mean by standard normal distribution?


A standard normal random variable Z has a normal distribution with mean =0 and standard
deviation =1.
Raw scores may be composed of large values, but large values cannot be accommodated
at the base line of the normal curve. So, they must be transformed into scores for convenience
without sacrificing meanings associated with the raw scores. If you wish to find the proportion
of area, percentage or probability associated with a raw score, you must find its matched z-value
using the z-score formula. Then, the z-value leads to the area under the normal curve found in the
z-table, which is a probability or the desired percentage.

Let’s explore the concept of z-scores.

Z-score or standard score measures how many standard deviation a given value (x) is above
or below the mean. Z-scores are useful in comparing observed values. If a z-score is equal to 0, it is
on the mean. A positive z-score indicates that the score or observed value is above the mean,
whereas a negative z-score indicates that the score or observed value is below the mean.

For example, if a z-score is equal


to 1, it is 1 standard deviation above the
mean. If a z-score is equal to −2, it is 2
standard deviations below the mean.
To solve the application problems
in this section, transform the values of the
variable to z values and then find the
areas under the standard normal
distribution. The areas under the normal
curve are given in terms of z-scores. Either it locates x within a sample or within a population.

39
The formula for calculating z is:

𝑋−𝜇
For Sample: 𝑧=
𝜎
𝑋− 𝑋̅
For Population: 𝑧 =
𝑠
where: z = standard score
Χ= raw score
𝑋̅ = sample mean
s = sample standard deviation
𝜇= population mean
𝜎 = population standard deviation

Study the following examples:

1. Choose the formula to use. (Use the z-


score formula for population data as 𝑋−𝜇
𝑧=
indicated in the problem 𝜎

2. Write the given values. X = 55; 𝜇 = 50; 𝜎 = 2


𝑋−𝜇
2. problem.) 𝑧=
𝜎
55 − 50 5
3. Substitute the given values in the = =
2 2
computing formula. Then, compute the 𝑧 = 2.5
z-value.

Example 1: Find the z-value that corresponds to Mathematics test score of 55 given the mean,
𝜇 = 50 and the standard deviation, 𝜎 = 2.
Solution:

Thus, the z-value that corresponds to the raw score 55 is 2.5 in a population distribution.
This means the score 55 is 2.5 standard deviations above the mean.

Example 2: On the midterm examination in Statistics, the sample mean was 80 and the sample
standard deviation was 7. Determine the standard score of a student who got a score of 75
assuming that the scores are normally distributed.
Solution:

𝑋 − 𝑋̅
1. Choose the formula to use. (Use 𝑧=
𝑠
the z-score formula for sample
data as indicated in the problem.)

2. Write the given values. ̅= 80; s= 7


Χ = 75; 𝑋

3. Substitute the given values in the 𝑋 − 𝑋̅


computing formula. Then, compute 𝑧=
𝑠
the z-value. 75 − 80 −5
= =
7 7

𝑧 = −0.71

29 40
Thus, the z-value that corresponds to the raw score 75 is −0.71 in a sample distribution.
This means the score 75 is 0.71 standard deviations below the mean.

Example 3. Cassey scored 78 on her History test which had a mean of 70 and a standard deviation of 3
and she scored 84 on her Math test which had a mean of 80 and a standard deviation of 2,
on which test did she score better? In which subject was her standing better, assuming
that the scores in her Mathematics and History class are normally distributed?

Solution:

1. Choose the formula to use. (For 𝑋−𝜇


𝑧=
population data) 𝜎

Subject Cassey mean Standard


Score (𝜇) deviation(𝜎)
2. Write the given values. History 78 70 3
Math 84 80 2

History Mathematics
s
𝑋−𝜇 𝑋−𝜇
𝑧= 𝑧=
3. Substitute the given values in the 𝜎 𝜎
computing formula. Then, 84 − 80
compute the z-value. 78 − 70 z=
z= 2
3
8 4
z= z=
3 2
𝑧 = 2.67 𝑧=2

The z-value that corresponds to Cassey’s History raw score of 78 is 2.67 in a population
distribution. This means the score 78 is 2.67 standard deviation above the mean. Meanwhile, the
z-value that corresponds to her Mathematics score of 84 is 2. This means that 84 is 2 standard
deviations above the mean. Thus, Cassey has a better standing in History compared to Mathematics

Let’s extend your learning!

You already know how to convert a random normal variable to a standard normal score
or z-score. This time let’s do the reverse. Given the z-score, compute for the raw scores.

1. Given: 𝜇 = 30, 𝜎 = 5. What is the raw score when 𝑧 = 1.30?


𝑥−𝜇
𝑧=
𝜎
1. Use the computing formula for finding the z-
score for population data. You can derive 𝑥−𝜇
the formula for easy computation. 𝜎(𝑧) = ( )𝜎
𝜎
𝑧𝜎 = 𝑥 − 𝜇
𝑥 = 𝜇 + 𝑧𝜎
2. Write the given values. 𝜇 = 30;𝜎 = 5, 𝑧 = 1.30

41
𝑥 = 𝜇 + 𝑧𝜎
= 30 + 1.30(5)
3 . Substitute the given values in the = 30 + 6.5
computing formula. Then, compute the raw 𝑥 = 36.5
score ( ).
Thus, the raw score when 𝐳 = 𝟏. 𝟑𝟎 is
36.5.

2 . Given: 𝜇 = 64=, 𝜎 = 7. What is the raw score when 𝑧 = 0.76?

𝑥−𝜇
𝑧=
𝜎
1. Use the computing formula for finding the z-score
𝑥−𝜇
for sample data. You can derive the formula for easy 𝜎(𝑧) = ( )𝜎
computation. 𝜎
𝑧𝜎 = 𝑥 − 𝜇
𝑥 = 𝜇 + 𝑧𝜎

2. Write the given values. 𝜇 = 64;𝜎 = 7, 𝑧 =0.76

3. Substitute the given values in the computing 𝑥 = 𝜇 + 𝑧𝜎


formula. Then, compute the raw score ( ). = 64 + 0.76(7)
= 64 + 5.32
𝑥 = 69.32

Thus, the raw score when 𝒛 = 𝟎. 𝟕𝟔 is


69.32

Let’s Summarize!

The formula for calculating z is:


𝑋−𝜇
For Sample: 𝑧=
𝜎

𝑋−𝑋̅
For Population: 𝑧 =
𝑠

• Z-score or standard score measures how many standard deviation a given value (x) is above
or below the mean.
• If a z-score is equal to 0, it is on the mean. A positive z-score indicates that the score or
observed value is above the mean, whereas a negative z-score indicates that the score
or observed value is below the mean.

II. LEARNING COMPETENCIES:


1. Converts a normal random variable to a standard normal variable and vice versa (MII/12SP-
IIIc4)
2. Solves word problems involving z-scores.

42
III. ACTIVITIES
ACTIVITY 1: COMPLETE ME
Complete the table below. Show your solution.

𝑿 𝝁 𝝈 𝒛
1 23 32 8 ?
2 250 231 120 ?
3 98 ? 5 2.21
4 ? 450 15 -1.5

Activity 2: WHAT I CAN DO


Directions: Calculate the z-score, given the mean, standard deviation, and raw
score (X).

1. Scores on a history test have an average of 80 with a standard deviation of 6. What is the z-score
for a student who earned a 75 on the test?
2. The weight of chocolate bars from a particular chocolate factory has a mean of 8 ounces with a
standard deviation of .1 ounce. What is the z-score corresponding to a weight of 8.17 ounces?
3. Books in the library are found to have an average length of 350 pages with a standard deviation
of 100 pages. What is the z-score corresponding to a book of length 80 pages?
4. The temperature is recorded at 60 airports in a region. The average temperature is 67 degrees
Fahrenheit with a standard deviation of 5 degrees. What is the z-score for a temperature of 68
degrees?

Activity 3: SOLVE ME
To solve the application problems in this section, transform the values of the variable to z
values.

1. Alex scored 90 during the first periodic exam in Mathematics and 88 during the second periodic
exam. The scores in first periodic exam have a mean 𝜇 = 83 and a standard deviation 𝜎 = 9. Scores
in the second periodic exam have a mean 𝜇 = 80 and a standard deviation 𝜎 = 8. In which
periodic exam was his standing better, assuming that the scores in his periodic exams are normally
distributed?
2. On a final examination in Biology, the mean was 75 and the standard deviation was 12. Determine
the standard score of a student who received a score of 60 assuming that the scores are normally
distributed.

III. REFLECTION

4. What have you learned from this activity?


_____________________________________________________________________
5. What did you like most from this lesson?
_____________________________________________________________________
6. What is the most challenging or least interesting from the activities?
____________________________________________________________________________

43
IV. REFERENCES

Belecina, Rene, E. Baccay, and E. Mateo (2016). Statistics and Probability: Rex Book Store, Inc., Manila
Philippines.
Glen, Stephanie "Normal Distributions (Bell Curve): Definition, Word Problems".
StatisticsHowTo.com: Elementary Statistics for the rest of us!Retrived from URL on May 22,.
Bluman, Allan G. (2012) Elementary Statistics:A Step by Step Approach, Eighth Edition, McGraw-Hill
Companies Inc., 1221 Avenue of the Americas, New York, pp.344-345
Practice Problems for Z-Scores (thoughtco.com)

VI. ANSWER KEY

in final examination in Biology is -1.25


score of 60 2. Standard score of students who received

compared to the first periodic exam.


better Alex’s standing the second periodic exam

exam: z 1
Second periodic First periodic exam: z =

Activity 3: SOLVE ME

4. z-score =0.2
3. z-score =-2.7
2. z-score =1.7
1. z-score =-0.83

Activity 2: WHAT I CAN DO

-1.5 15 450 427.5 4


2.21 5 86.95 98 3
0.1583 120 231 250 2
-1.125 8 32 23 1
𝒛 𝝈 𝝁 𝑿

Activity 1: COMPLETE ME

Prepared by:

CHARMIE G. RELLORES
Writer

44
STATISTICS AND PROBABILITY

Name of Learner: _____________________________ Section: _________________


Grade Level: _________________________________ Date: ___________________

Q3_W4_LAS8
COMPUTING PROBABILITIES AND PERCENTILES UNDER THE NORMAL CURVE

I. BACKGROUND INFORMATION FOR LEARNERS


You have already learned about finding the areas under the normal curve. Further, you also
learned how to compute the z-value corresponding to a raw score. You have also seen that all areas
under the normal curve can be shown as probabilities associated with standard normal variables. In this
lesson, you will practice more your skills in determining areas under the normal curve since computing
probabilities is the same with finding the areas under the normal curve.

Finding the area of a region under the normal curve is the same as finding the probability
associated with that region. Thus, the area under the normal curve denotes probability. Hence, we will
find the probability between two z-values by simply calculating the required area.

Probability Notations:

Let a and b be z-score values:

• 𝑃(𝑎 < 𝑧 < 𝑏) denotes the probability that the z-score is between a and b. It is read as “the
probability that the z-score falls between 𝑧 = 𝑎 and 𝑧 = 𝑏.”

• 𝑃(𝑧 > 𝑎) denotes the probability that the z-score is greater than a. It is read as “the probability
that the z-score is greater than a.”

• 𝑃(𝑧 < 𝑎) denotes the probability that the z-score is less than a. It is read as “the probability
that the z-score is less than a.”

Recall each case in identifying regions under the curve and the steps on determining the area
under the normal curve.

Example 1: Find the probability that the z-score is at most 𝑧 = 0.91

1. Draw the normal curve and locate the given z-


value or values at the base line of the curve.
Then, draw a vertical line through the given z
value or values and shade the required region.

2. Use the z-table to find the areas that 𝑧 = 0.91 corresponds to an area of 0.3186
correspond to the given z-value or values.

3. Perform appropriate operations to get the The graph suggests subtraction. 0.3186 + 0.5 =
required area, if needed. 0.8186 That is, 𝑷(𝒛 < 0.91) = 𝟎. 𝟖𝟏𝟖𝟔.

4. Write the required area (or probability). Thus, the probability that the z-score is at most𝑧
− 1.78 is 0.8186.

45
Example 2: Find the probability that the z-score is less than 𝑧 = −1.78.

1. Draw the normal curve and locate the given z-


value or values at the base line of the curve.
Then, draw a vertical line through the given z-
value or values and shade the required region.

2. Use the z-table to find the areas that 𝑧 = −1.78 corresponds to an area of 0.4625
correspond to the given z-value or values.

3. Perform appropriate operations to get the The graph suggests subtraction. 0.5 − 0.4625 =
required area, if needed. 0.0375 That is, 𝑷 (𝒛 < −1.78) = 𝟎. 𝟎𝟑𝟕𝟓.

4. Write the required area (or probability). Thus, the probability that the z-score is less
than𝑧 − 1.78 is 0.0375.

What is a percentile?

Often the units for raw test scores are not informative. You might know that you scored 90 out
of 100 on a test but it doesn’t give you much information of what your score means. Scores are more
meaningful if percentile rank is given. What do you mean by a percentile? If your teacher told you that
your score in test is in the 90th percentile. What does it mean?

Percentile is the score at which a specified percentage of scores in a distribution fall below.
Percentile is a measure of relative standing. It is the percent of cases that are at or below a score. It
tells you how a value compares to other values.

➢ If your teacher tells you that you scored 90th percentile, it means that 90% of the grades were lower than
yours and 10% were higher.
➢ To say a score of 53 is in the 75th percentile is to say that 75% of all scores are less than 53.

In the previous lesson, you learned on how to find the area or probability under the normal
curve given a z-value. Suppose, you know the area or probability this time, how would you find the
corresponding z-score?

When you are given with the area or probability and you want to know the corresponding
z-score, locate the area at the body of the table. If the exact area is not available, take the nearest
area. Then, look up the corresponding z-value.

Example 1: Find the 90th percentile of a normal curve. Finding 90 th percentile means locating an area
below the point. To begin, find the z-value located at this point.

Solution:
Step 1: Express the 90th percentile to decimal so that you can easily find it in the z
table, that is, 0.9000. We know that 0.9000 occupies more than half of the
curve. This occupies all the area below the mean which is 0.5 or 0.5000. This
left us with an area of 0.4000 above the mean (0.9000 - 0.5000 = 0.4000).

46
Step 2: Locate the z-value corresponding to the area 0.4000. The z-value corresponding
to the area 0.4000 is not available, so take the nearest area. The nearest area is
0.3997 which correspond to z = 1.28. So, the 90th percentile is z=1.28

Step 3: To illustrate, draw the normal curve. Draw a line through 𝑧 = 1.28 and shade the
region below it. The shaded region is 90% of the distribution.

Example 2: Find the 95th percentile of a normal curve. Finding 95thpercentile means locating an area
below the point. To begin, find the z-value located at this point.

Solution:
Step 1: Express the 95th percentile to decimal so that you can easily find it in the z
table, that is, 0.9500. We know that 0.9500 occupies more than half of the
curve. This occupies all the area below the mean which is 0.5 or 0.5000. This
left us with an area of 0.4500 above the mean (0.9500-0.5000=0.4500).

Step 2: Locate the z-value corresponding to the area 0.4500. The z-value corresponding
to the area 0.4500 is not available, so take the nearest area. The nearest areas
are 0.4495 and 0.4505. Find the z-value corresponding to 0.4495 and 0.4505.
These are z=1.64 and z=1.65. We get the average of the two z-values:
𝑧 = 1.64+1.65 2 = 𝟏. 𝟔𝟒𝟓.Thus, the 95th percentile is z=1.645.

Step 3: To illustrate, draw the normal curve. Draw a line through 𝑧 = 1.645 and shade
the region below it. The shaded region is 95% of the distribution.

Example 3: Find the upper 2% of the normal curve. Finding the upper 2% of the normal curve means
locating an area above the point. To begin, find the z-value located at this point.

Solution:
Step1: Express the given percent to decimal so that you can easily find it in the z table, that
is, 0.0200. The upper 5% or 0.0500 means to the right of a z-value above the mean.
Find the remaining area, using the upper side of the mean (0.5000- 0.0200=0.4800).

Step 2: Locate the z-value corresponding to the area 0.4800. The z-value corresponding to
the area 0.4800 is not available, so take the nearest area. The nearest area is 0.4798
which corresponds to 𝑧 = 2.05. Thus, the upper 2% is above 𝒛 = 𝟐. 𝟎𝟓.

47
Step 3: To illustrate, draw the appropriate normal curve. Draw a line through 𝑧 = 2.05 and
shade the region above it. The shaded region is 2% of the distribution.

Let’s Summarize!
Finding the area of a region is the same as finding the probability associated with that region.

• The following are probability notations used to denote probabilities under the normal curve.

1. 𝑃(𝑎 < 𝑧 < 𝑏)denotes the probability that the z-score is between a and b.

2. 𝑃(𝑧 > 𝑎)denotes the probability that the z-score is greater than a.

3. 𝑃(𝑧 < 𝑎)denotes the probability that the z-score is less than a.

• Percentile is a measure of relative standing. It is the percent of cases that are at or below a score.
It tells you how a value compares to other values.

• When you are given with the area or probability and you want to know the corresponding z-
score, locate the area at the body of the table. If the exact area is not available, take the nearest
area. Then, look up the corresponding z-value.

II. LEARNING COMPETENCY


Computes probabilities and percentiles using standard normal table (MII/12SP-IIIc-d-1).

III. ACTIVITIES
ACTIVITY 1: WHAT I CAN DO?
Find the probabilities for each, using the standard normal distribution.

1. 𝑃(0 < 𝑧 < 1.96)


2. 𝑃(𝑧 > −1.43)
3. 𝑃(1.12 < 𝑧 < 1.43)
4. 𝑃(−1.23 < 𝑧 < 0)

ACTIVITY 2: FIND ME!


Directions: Find each of the following percentile points and draw the appropriate normal curve.
Complete your procedures.

1. Find the 99th percentile of the normal curve.


2. Find the upper 5% of the normal curve.
3. Find the 92nd percentile rank of the normal curve.

48
IV. REFLECTION

1. What have you learnt from this activity?


_____________________________________________________________________

2. What did you like most from this lesson?


_____________________________________________________________________

3. What is the most challenging from the activities?


_________________________________________________________________________
_______________________________________________________________________

V. REFERENCES:

Belecina, Rene, E. Baccay, and E. Mateo (2016). Statistics and Probability: Rex Book Store, Inc., Manila
Philippines.
Laerd Statistics. “How to do Normal Distributions Calculations”. Accessed May 25, 2020.
https://statistics.laerd.com/statistical-guides/normal-distributioncalculations.
Bluman, Allan G. (2012) Elementary Statistics:A Step by Step Approach, Eighth Edition, McGraw-Hill
Companies Inc., 1221 Avenue of the Americas, New York, pp.347-353

VI. ANSWER KEY

3. The 92nd percentile rank is z= 1.41


2. The upper 5% of the normal curve is above 𝑧 = 1.645.
1. The 99th percentile is 𝑧 = 2.33.
Activity 2: Find Me!

4. 𝑃(−1.23 < 𝑧 < 0) − 0.3907


3. 𝑃(1.12 < 𝑧 < 1.43) − 0.0550
2. 𝑃(𝑧 > −1.43) − 0.9236
1. 𝑃(0 < 𝑧 < 1.96) − 0.4750

Activity 1: What I Can Do?

Prepared by:

CHARMIE G. RELLORES
Writer

49
STATISTICS AND PROBABILITY

Name of Learner: _____________________________ Section: _________________


Grade Level: _________________________________ Date: ___________________

Q3_W5_LAS9
ILLUSTRATING RANDOM SAMPLING

I. BACKGROUND INFORMATION FOR LEARNERS


This activity sheet in Statistics and probability is designed for you Grade 11 learners. The
activities that you will perform are organized and made simple for you to exercise independent learning.
In this particular Learning Activity Sheet (LAS), you will be able to illustrate random sampling.
Before answering the series of activities, take time to read the concepts that you need to know.

Random Sampling

In our everyday life, we are oriented with basic concepts in statistics. When you want to
know the taste of food your mother is cooking, you only taste only a spoonful of it. On a wider
scope, researchers aim to study, describe and infer patterns of behavior, properties and
characteristics about a population. It is impractical and inconvenient to study the whole population
that is why samples are only selected as representatives.
A population is a group where members have something in common, that is, the total set
of observations that can be made. Some examples include the population of women aged 20 and
above, the population of registered voters in the province of Albay, and the population of daily
maximum temperatures for the month of April in Baguio City. A sample is a smaller group or subset
of the population that you want to examine.
Sampling is a process used in statistical analysis in which a predetermined number of
observations are taken from a larger population. When each element of the population has an
equal chance of being selected, it is called random sampling.
There are different sampling methods that allow all the units in the population to have an
equal chance of being selected.

50
1. Simple Random/Lottery Sampling – a sampling technique by which every member of the
population has an equal chance to be chosen as sample. It can be done through lottery method,
table of random numbers or through the use of computers.

2. Systematic Random Sampling – a sampling technique by which every member of the


population is selected with a random start.

For example, in 240 grade 11 students, you can only choose 80 students as members of
your sample. To select which students will be part of the sample, we just divide the population
size 240 by the sample size 80, which results to 3. That means every 3rd student among the 240
grade 11 students will be included in the sample. The researcher may also opt to start on any
randomly selected number from 1 to 240 then select every 3rd element successively until the
desired sample size is completed.

51
3. Stratified Random Sampling – a sampling technique that is used when the population can be
classified into groups or strata based on some characteristics. Common criterions used for
stratification are gender, age, ethnicity, and socioeconomic status.

4. Clustered Sampling – a sampling method where the population is first divided into separate
groups called clusters. Then a simple random sample of clusters from the available clusters in the
population is selected. This sampling technique is usually used when the elements of a population
are spread over a wide geographical area.

For example, if the population is composed of all citizens residing in the Philippines, the
clusters could be citizens from Luzon, Visayas, and Mindanao. Data is then gathered from a
selected cluster.

II. LEARNING COMPETENCY


Illustrates random sampling. (M11/12SP-IIIc-2)

52
III. ACTIVITIES
ACTIVITY 1: RANDOM SAMPLE OR NOT?

Determine whether the process of getting a sample in each situation is by random sampling or not.
1. To determine the common T-shirt size his students have, Mr. Santos draw his sample from a bowl
containing the names of his students with their T-shirt sizes.
Answer: ______________
2. To determine the most liked subject in their school, Sarah interviewed the president of each class.
Answer: ______________
3. To know the most preferred learning modality of his classmates, Joseph interviewed their class
officers.
Answer: ______________
4. To determine the performance of of the SHS students in Statistics, the teacher draws 5 students
from every SHS class to take the Statistics test.
Answer: ______________

ACTIVITY 2: IDENTIFY ME!


Identify the type of random sampling illustrated by the following situations.

1. A barangay chairman wants to know whether his constituents are in favor or not on putting
satellite markets in their barangay. He wants to select a sample of 200 from his constituents,
from the youths, adults and from the senior citizens.
Answer: ______________
2. A researcher selected the participants of his study by selecting every 5th member of the
population.
Answer: ______________
3. A researcher used his computer in randomly selecting a sample of n = 240 from a population of
800.
Answer: ______________
4. A researcher surveyed all teachers in each of the 12 randomly selected secondary schools in
Albay.
Answer: ______________

53
ACTIVITY 3: THINK ABOUT IT!
Give one research situation where each of the random sampling methods is being applied.
1. Simple Random Sampling
______________________________________________________________________________
______________________________________________________________________________
______________________________________________________________________________

2. Systematic Random Sampling


______________________________________________________________________________
______________________________________________________________________________
______________________________________________________________________________

3. Stratified Random Sampling


______________________________________________________________________________
______________________________________________________________________________
______________________________________________________________________________

4. Cluster Sampling
______________________________________________________________________________
______________________________________________________________________________
______________________________________________________________________________

IV. REFLECTION:
1. What are your learnings from the activites?
___________________________________________________________________________________
___________________________________________________________________________________

2. What part of the lesson did you like most?


___________________________________________________________________________________
___________________________________________________________________________________

3. What challenges did you encounter from the activities?


___________________________________________________________________________________
___________________________________________________________________________________

54
V. REFERENCES

Belecina, Rene R., Elisa S. Baccay, and Efren B. Mateo (2016). Statistics and Probability, Manila: Rex
Book Store Inc., 101-131

Bluman, A.G. (2012). Elementary Statistics: A Step-by-Step Approach, 8th Ed. The McGraw-Hill
Companies, Inc.

Febri, Francisco Jr. (1987). Introduction to Statistics. Quezon City: Phoenix Publishing House, Inc. 85-101

https://faculty.elgin.edu/dkernler/statistics/ch01/1-4.html

4. Clustered random sampling


3. Simple random sampling
2. Systematic random sampling
1. Stratified random sampling
Activity 2: Identify Me!

4. Random sample
3. not
2. not
1. Random sample
Activity 1: Random Sample or Not?

Answer Key:

Prepared by:
ARIS B. NISOLA
Writer

55
STATISTICS AND PROBABILITY

Name of Learner: _____________________________ Section: _________________


Grade Level: _________________________________ Date: ___________________

Q3_W5_LAS10
DISTINGUISHING PARAMETER AND STATISTIC

I. BACKGROUND INFORMATION FOR LEARNERS


In the previous activity, you learned about random sampling, a process done in selecting an
unbiased representative sample from a population.
In this particular Learning Activity Sheet (LAS), you will be able to identify and distinguish
measures about population and sample called parameter and statistic. Take time to read the examples
below for you to be able to answer the succeeding activities.

Parameter and Statistic

A measure that describes a population is called a parameter. A measure that describes a


sample is called statistic. There is a simple and straightforward way to remember what a
parameter and statistic are measuring. All that we must do is look at the first letter of each word.
A parameter measures something in a population, and a statistic measures something in a sample.

Examples:
Parameter: the average IQ obtained from the entire population of grade 11 students
Statistic: the average IQ obtained from a sample of grade 11 students

Parameter: The average weekly allowance of all Grade 11 male students


Statistic: The average weekly allowance of a sample of 60 male students

Parameter: The average time spent by all the teachers at a certain school in sorting Self-Learning
Modules is 2.2 hours.
Statistic: A survey revealed that from the randomly selected sample of 400 high school students
in the Albay Division, 75% do not have internet connection at home.

In population parameter, mean is represented by μ, σ2 represents variance, and σ represents


standard deviation. In sample statistics, sample mean is represented by 𝑥̅ , s2 represents sample
variance and s represents sample standard deviation.

56
II. LEARNING COMPETENCY

The learner distinguishes between parameter and statistic. (M11/12SP-IIIc-3)

III. ACTIVITIES

ACTIVITY 1: TRUE OR FALSE?


Determine whether the statement is true or false. Write T if the statement is true and F if
false on the blank provided before each item.

____1. The given value in “The average score of all the TVL students in Statistics and Probability
of a certain school is 83%.” is a parameter.
____2. An example of a parameter is s2.
____3. The descriptive measure computed from the entire population of data is called
parameter.
____4. The average age of all the grade 11 students is an example of statistic.

ACTIVITY 2: PARAMETER OR STATISTIC?


Identify whether the given value is a parameter or a statistic.

_____________1. A researcher found out that the average salary of all the employees in a certain
company is Php 16,240.
_____________2. The average score of 36 out of 150 Grade 11 students in an examination is 46.
_____________3. Based on a sample of 400 students, it was found out that 78% of them do not have
gadgets for online learning.
_____________4. The teacher surveyed all 48 students in her advisory class about their preferred
distance learning delivery modality and found out that 83% of them wanted
modular learning.
ACTIVITY 3: WHICH IS WHICH?
Read the statistical study in each item. For each study, identify both the parameter and the
statistic. Refer to the example below.
Example: A researcher wants to determine the average time spent by grade 11 students in reading
answering Statistics modules. From a random sample of 60 grade 11 students, he obtained a sample
mean time of 4.2 hours.
Parameter: The average time spent by all grade 11 students in reading and answering Statistics
modules
Statistic: The average time of 4.2 hours from a sample of 60 grade 11 students

57
1. A teacher wants to determine the average age of grade 11 students. From a random sample of 40
students, the teacher obtained an average age of 17.
Parameter: __________________________________________________________________
Statistic: ____________________________________________________________________

2. A teacher wants to know the average time spent in playing online games of his advisory class. He
surveyed a random sample of 30 students and found out that they spend an average of 4 hours per
day in playing online games.
Parameter: __________________________________________________________________
Statistics: ___________________________________________________________________

3. A researcher wants to determine the average time spent by Statistics teachers in preparing learning
activity sheets. She randomly selected 50 teachers and obtained an average time of 2.5 hours.
Parameter: __________________________________________________________________
Statistic: ____________________________________________________________________

4. A researcher wants to determine the mean height of grade 11 male students. He obtained a mean
height of 154 cm from the 60 male students he randomly selected.
Parameter: __________________________________________________________________
Statistic: ____________________________________________________________________

IV. REFLECTION
1. What have you learned from this activity?
__________________________________________________________________________________
__________________________________________________________________________________
__________________________________________________________________________________
2. What part of the lesson did you like most?
__________________________________________________________________________________
__________________________________________________________________________________
__________________________________________________________________________________
3. What challenges did you encounter from the activities?
__________________________________________________________________________________
__________________________________________________________________________________
__________________________________________________________________________________

58
59
Writer
ARIS B. NISOLA
Prepared by:
Answer Key:
Activity 1: True or False?
1. T
2. F
3. T
4. F
Activity 2: Paramter or Statistic?
1. Parameter
2. Statistic
3. Parameter
4. Statistic
Activity 3: Which is which?
1. Parameter: The average age of all grade 11 students
Statistic: The average age of 17 from a random sample of 40 students
2. Parameter: The average time spent by all his students in playing online games
Statistic: The average time of 4 hours per day spent by a random sample of 30 students in
playing online games
3. Parameter: The average time spent by all Statistics teachers in preparing learning activity
sheets
Statistic: The average time of 2.5 hours spent by 50 randomly selected teachers in preparing
learning activity sheets
4. Parameter: The mean height of all grade 11 male students
Statistic: The mean height of 154 cm of the 60 randomly selected grade 11 male students
https://faculty.elgin.edu/dkernler/statistics/ch01/1-4.html
101
Febri, Francisco Jr. (1987). Introduction to Statistics. Quezon City: Phoenix Publishing House, Inc. 85-
Companies, Inc.
Bluman, A.G. (2012). Elementary Statistics: A Step-by-Step Approach, 8th Ed. The McGraw-Hill
Book Store Inc., 101-131
Belecina, Rene R., Elisa S. Baccay, and Efren B. Mateo (2016). Statistics and Probability, Manila: Rex
V. REFERENCES
STATISTICS AND PROBABILITY

Name of Learner: _____________________________ Section: _________________


Grade Level: _________________________________ Date: ___________________

Q3_W5_LAS11
IDENTIFYING SAMPLING DISTRIBUTIONS OF STATISTICS (SAMPLE MEAN)

I. BACKGROUND INFORMATION FOR LEARNERS


You learned in the previous chapters about discrete probability distribution and continuous
probability distribution. In this activity, you will learn how to identify and construct sampling
distribution of the sample means. Are you ready?
Take time to read the discussion and procedure on how to construct sampling distribution of
the sample means.

Warm-Up

One of the prerequisite skills that you need to have in this activity is for you to compute
for the mean of a given set of numbers. Can you find the mean of the sets of numbers below?
1) 2, 4, 7, 9 2) 3, 10, 5, 8, 4 3) 9, 10, 12, 15, 18, 20

If your answers are: 5.5, 6 and 14 for items 1, 2 and 3 respectively, then you did a great
job in finding the mean of the given set of numbers!

List Them All!

Suppose a population consists of these numbers: 2, 4, 8 and 10. Can you list all the possible
samples of size 2 which can be drawn from this population? One possible sample is shown below.
Sample
2, 4

60
Were you able to identify all samples of size 2? Check your answers by referring to the list
below.
Sample
2, 4
2, 8
2, 10
4, 8
4, 10
8, 10

Mean-ie Me!

Now that you were able to list all the possible samples of size 2 from the population
containing the numbers 2, 4, 8 and 10, it’s time to compute for the mean of each sample.

Sample Mean
2, 4 3
2, 8 5
2, 10 6
4, 8 6
4, 10 7
8, 10 9

This let us learn how to make a probability distribution of the sample means, which is
called the sampling distribution of sample means. Study the table below.

Sample Mean Probability


Frequency
̅
𝒙 ̅)
P(𝒙
3 1 1
6
5 1 1
6
6 2 1
3
7 1 1
6
9 1 1
6

61
Notice that the first column contains all the possible sample means. The second column
contains the frequency of each sample mean. Take note that in the previous activity, two samples
of size 2 have a mean of 6. Finally the third column contains the probabilities of the possible
sample means.

The probability distribution of the sample means is also called the sampling distribution
of the sample means. This describes the probability for each mean of all samples with the same
sample size n.

II. LEARNING COMPETENCY


The learner identifies sampling distributions of statistics (sample mean) – M11/12SP-IIIc-4

III. ACTIVITIES

ACTIVITY 1: LIST AND CONSTRUCT!


A population consists of the five numbers 2, 5, 6, 8, and 11. Consider samples of size 2 that
can be drawn from the population.
a. List all the possible samples and the corresponding mean.
Sample Mean

62
b. Construct the sampling distribution of the sample means.

Sample Mean Probability


Frequency
̅
𝒙 ̅)
P(𝒙

ACTIVITY 2: TEST SCORES

A group of students got the following scores in a test: 6, 9, 12, 15, and 21. Consider samples
of size 3 that can be drawn from this population.

a. List all the possible samples and the corresponding mean.

Sample Mean

63
b. Construct the sampling distribution of the sample means.

Sample Mean Probability


Frequency
̅
𝒙 ̅)
P(𝒙

ACTIVITY 3: DRAWING CARDS


Samples of 4 cards are drawn from a population of 6 cards numbered 1-6.

Construct a sampling distribution of the sample means and answer the following questions:

1. How many samples of size 4 can be drawn from the population?


2. What are the possible means?
3. What is the probability of getting 4 as a mean?
4. What is the probability of getting 3.5 as a mean?

(You may use an extra sheet of paper for this activity.)

64
IV. REFLECTION
1. What have you learned from this activity?
__________________________________________________________________________________
__________________________________________________________________________________
__________________________________________________________________________________

2. What part of the lesson did you like most?


__________________________________________________________________________________
__________________________________________________________________________________
__________________________________________________________________________________

3. What challenges did you encounter from the activities?


__________________________________________________________________________________
__________________________________________________________________________________
__________________________________________________________________________________

VI. REFERENCES
Belecina, Rene R., Elisa S. Baccay, and Efren B. Mateo (2016). Statistics and Probability, Manila: Rex
Book Store Inc., 101-109

Bluman, A.G. (2012). Elementary Statistics: A Step-by-Step Approach, 8th Ed. The McGraw-Hill
Companies, Inc.

Febri, Francisco Jr. (1987). Introduction to Statistics. Quezon City: Phoenix Publishing House, Inc. 85-
101

https://faculty.elgin.edu/dkernler/statistics/ch01/1-4.html

Prepared by:
ARIS B. NISOLA
Writer

65
66
Answer Key:
Activity 1: List and Construct
a. Activity 2: Test Scores
Sample Mean a.
2, 5 3.5 Sample Mean
2, 6 4 6, 9, 12 9
2, 8 5 6, 9, 15 10
2, 11 6.5 6, 9, 21 12
5, 6 5.5 6, 12, 15 11
5, 8 6.5 6, 12, 21 13
5, 11 8 6, 15, 21 14
6, 8 7 9, 12, 15 12
6, 11 8.5 9, 12, 21 14
8, 11 9.5 9, 15, 21 15
12, 15, 21 16
b.
Sample Probability b.
Frequency Sample Probability
Mean P(𝒙
̅) Frequency
̅
𝒙 Mean P(𝒙
̅)
3.5 1 1/10 𝒙̅
4 1 1/10 9 1 1/10
5 1 1/10 10 1 1/10
5.5 1 1/10 11 1 1/10
6.5 2 1/5 12 2 1/5
7 1 1/10 13 1 1/10
8 1 1/10 14 2 1/5
8.5 1 1/10 15 1 1/10
9.5 1 1/10 16 1 1/10
Activity 3: Drawing Cards
Sample Probability
Frequency
Mean P(𝒙
̅)
̅
𝒙
2.5 1 1/15
2.75 1 1/15
3 2 2/15
3.25 2 2/15
3.5 3 1/5
3.75 1 1/15
4 3 1/5
4.25 1 1/15
4.5 1 1/15
Answers to the questions:
1) 15 3) 1/5
2) 2.5, 2.75, 3, 3.25, 3.5, 3.75, 4, 4.25, and 4.5 4) 1/5
STATISTICS AND PROBABILITY

Name of Learner: _____________________________ Section: _________________


Grade Level: _________________________________ Date: ___________________

Q3_W6_LAS12
FINDING THE MEAN AND VARIANCE OF SAMPLING DISTRIBUTION OF THE SAMPLE MEANS

I. BACKGROUND INFORMATION FOR LEARNERS


Hello dear young statistician! You are now on the 6 th week of Quarter 3. In the previous lesson
you have learned several ways to select a random sample from a population.

This week, we will be exploring the Mean and Variance of Sampling Distribution of the Sample
Means. In this lesson, you will learn how to describe the sampling distribution of the sample means
using the means and variance.

Now, it’s time for you to explore and learn. Good luck!

• The probability distribution of the sample means is also called the sampling distribution of the
sample means.
• A sampling distribution of sample means is a frequency distribution that describes the
probability for each mean of all samples with the same sample size n.
• The standard deviation of the sampling distribution of the sample means is also
known as the standard error of the mean. It measures the degree of accuracy of the sample
mean (x̄) as an estimate of the population mean (µ).
• When taking samples of size n from any population with finite mean µ and variance σ2, then
the sampling distribution of the sample means has the following properties:
1. The mean of the sampling distribution of the sample means (µx̄ ) is equal to the population
mean ( µ ).
That is, µx̄ = µ
2. The variance of the sampling distribution of the sample means (σ2 x̄ ) is smaller than the
population
distribution, which is given by,
𝝈𝟐
✓ σ2 x̄ = ( with replacement)
𝒏
𝝈𝟐 𝑵− 𝒏
✓ σ2 x̄ = • ( without replacement)
𝒏 𝑵− 𝟏
3. The standard deviation of the sampling distribution of the sample means is equal to the
population standard
deviation divided by the square root of the sample size of n of the samples. That is,
𝛔
✓ σ x̄ = ( with replacement)
√𝐧
𝛔 𝐍− 𝐧
✓ σ x̄ = •√ ( without replacement )
𝒏 𝐍− 𝟏

67
• Formula’s in Computing the Mean, Variance, and Standard Deviation

SAMPLING DISTRIBUTION OF SAMPLE


POPULATION
MEANS
∑𝐗
MEAN µ = µ x̄ = ∑ 𝒙̄ • 𝑷 ( 𝒙̄ )
𝐍

∑( 𝑿− µ )𝟐
VARIANCE σ2 = σ2 x̄ = ∑ 𝐏(𝐱̄̄) • ( 𝐱̄̄ − µ )𝟐
𝑵

STANDARD ∑( 𝑿− µ )𝟐
DEVIATION σ = √ σ x̄ = √∑ 𝐏(𝐱̄̄) • ( 𝐱̄̄ − µ )𝟐
𝑵

In this learning activity sheet, you will learn how to describe the sampling distribution of the
sample means by computing its mean and variance.

II. LEARNING COMPETENCIES


1. Find the Mean and Variance of the Sampling Distribution of the Sample Mean (M11/12SP-IIId-
5)
2. Define the sampling distribution of the sample mean for normal population when the variance
is (a) known; and (b) unknown (M11/12SP-IIIe-1)

Example 1: SAMPLING without REPLACEMENT from a FINITE POPULATION

Consider a population consisting of the values 1, 2, 3, 4, and 5. Suppose samples of size 2 are
drawn from without replacement from the population. Describe the sampling distribution of the
sample means.

a) Compute the mean, variance, and standard deviation of the population.


b) Find the mean, variance, and standard deviation of the sampling distribution of the sample means.

STEPS SOLUTION

1. Compute the mean of the population (µ). ∑𝐗 𝟏+𝟐+ 𝟑+ 𝟒+ 𝟓


µ= = = 3.00
𝐍 𝟓
Hence, the mean of the population is 3.00

2. Compute the variance of the population (σ). X X-µ (X-µ)2

1 -2 4
2 -1 1
3 0 0
4 1 1
5 2 4

∑( 𝑿 − µ ) 2 = 10

∑( 𝑿− µ ) 𝟐 𝟏𝟎
σ2 = = = 2
𝑵 𝟓
Hence, the variance of the population is 2.

68
σ = √𝛔𝟐 = √𝟐 = 1.41
And the population standard deviation is 1.41.
Use the formula N C n where N = 5 and n = 2
3. Determine the number of possible 5 C 2 = 10
samples of size n = 2 Hence, there are 10 possible samples of size 2
that can be drawn.
Samples Mean
4. List all possible samples and them
corresponding means. 1 ,2 1.50
1 ,3 2.00
1 ,4 2.50
1 ,5 3.00
2 ,3 2.50
2 ,4 3.00
2 ,5 3.50
3 ,4 3.50
3 ,5 4.00
4 ,5 4.50

5. Construct the sampling distribution of the Sampling Distribution of Sample Means


sample means.
Sample
Probability
Mean Frequency
P ( x̄ )

1
1.50 1
10
1
2.00 1
10
1
2.50 2
5
1
3.00 2
5
1
3.50 2
5
1
4.00 1
10
1
4.50 1
10
TOTAL 10 1.00

69
6. Compute the mean (µ x̄ ) of the sampling Sample Probabilit
distribution of the sample means. Mean y x̄ • P ( x̄ )
Follow these steps: x̄ P ( x̄ )
a. Multiply the sample mean by the
1
corresponding probability. 1.50 0.15
10
b. Add the results.
1
2.00 0.20
10

1
2.50 0.50
5

1
3.00 0.60
5

1
3.50 0.70
5

1
4.00 0.40
10

1
4.50 0.45
10
∑ 𝒙̄ • 𝑷 ( 𝒙̄ )
TOTAL 1.00
= 3.00

µ x̄ = ∑ 𝒙̄ • 𝑷 ( 𝒙̄ )
= 3.00

Hence, the mean of the sampling distribution


of the sample
means is 3.00

7. Compute the variance (σ2 x̄ ) of the sampling P ( x̄ x̄ - (x̄ - µ P ( x̄ ) • (x̄ -


distribution of the sample means. x̄
) µ x̄ x̄ )
2 µ x̄ ) 2
Follow these steps: 1.5 1
a. Subtract the population mean (µ) from -1.50 2.25 0.225
0 10
each sample mean (x̄). Label this as 2.0 1
x̄ - µ -1.00 1.00 0.100
0 10
b. Square the difference. Label this as 2.5 1
( x̄ − µ ) 2 0.50 0.25 0.050
0 5
c. Multiply the results by the 3.0 1
corresponding probability. Label this as 0.00 0.00 0.000
0 5
P ( x̄ ) • ( x̄ − µ ) 2 3.5 1
d. Add the results. 0.50 0.25 0.050
0 5
4.0 1
1.00 1.00 0.100
0 10
70
4.5 1
1.50 2.25 0.225
0 10
TO
1.00 0.75
TAL

σ2 x̄ = ∑ 𝐏(𝐱̄̄) • ( 𝐱̄̄ − µ𝐱̄̄ )𝟐


= 0.75
Hence, the variance of the sampling
distribution is 0.75.

σ x̄ = √∑ 𝐏(𝐱̄̄) • ( 𝐱̄̄ − µ )𝟐
= √𝟎. 𝟕𝟓
= 0.87
And the standard deviation of the sampling
distribution is 0.87.

Example 2: SAMPLING with REPLACEMENT from a FINITE POPULATION

Consider a population consisting of the values 2, 5, and 8. List all the possible samples of size
n = 2 which can be drawn with replacement from the population. Describe the sampling distribution
of the sample means.

a) Compute the mean, variance, and standard deviation of the population.


b) Find the mean, variance, and standard deviation of the sampling distribution of the
sample means.

STEPS SOLUTION

1. Compute the ∑𝐗 𝟐+ 𝟓 + 𝟖
µ = = = 5.00
𝐍 𝟑
population mean (µ).
Hence, the population mean is 5.00

2. Compute the
X X-µ (X-µ)2
population
variance (σ2) 2 -3 9
and 5 0 0
population
8 3 9
standard
deviation (σ). ∑( 𝑿 − µ ) 2 = 18

∑( 𝑿− µ ) 𝟐 𝟏𝟖
σ2 = = = 6
𝑵 𝟑
Hence, the population variance is 6.

σ = √𝛔𝟐 = √𝟔 = 2.45
And the population standard deviation is 2.45.
Mean
3. List all Samples

possible
2 ,2 2.0
samples of size 2
, with 2 ,5 3.5

71
replacement 2 ,8 5.0
and their
5 ,2 3.5
corresponding
means. 5 ,5 5.0
5 ,8 6.5
8 ,2 5.0
8 ,5 6.5
8 ,8 8.0

Sampling Distribution of Sample Means


4. Construct the Sample Mean Probability
sampling Frequency
x̄ P ( x̄ )
distribution of 1
the sample 2.00 1
9
means. 2
3.50 2
9
3
5.00 3
9
2
6.50 2
9
1
8.00 1
9
TOTAL 9 1.00

5. Compute the Sample Mean Probability


x̄ • P ( x̄ ) µ
mean (µ x̄ ) of x̄ P ( x̄ )
x̄ =
the sampling
distribution 1
2.00 0.22
of the sample 9
means. 2
3.50 0.78
9
3
5.00 1.67
9
2
6.50 1.44
9
1
8.00 0.89
9
TOTAL 1.00 ∑ 𝒙̄ • 𝑷 ( 𝒙̄ ) = 5
∑ 𝒙̄ • 𝑷 ( 𝒙̄ ) = 5.00
Hence, the mean of the sampling distribution of the sample
means is 5.

6. Compute the P ( x̄ ) • (x̄ -


x̄ P ( x̄ ) x̄ - µ x̄ (x̄ - µ x̄ ) 2
variance (σ2 x̄ ) µ x̄ ) 2
and the 1
2.00 -3.00 9.00 1.00
9
72
standard 2
deviation (σ x̄ ) 3.50 -1.50 2.25 0.50
9
of the sampling
distribution of 3
5.00 0.00 0.00 0.00
the sample 9
means.
2
6.50 1.50 2.25 0.50
9
1
8.00 3.00 9.00 1.00
9
TOTAL 1.00 3

σ2 x̄ = ∑ 𝐏(𝐱̄̄) • ( 𝐱̄̄ − µ𝐱̄̄ )𝟐


= 3

Hence, the variance of the sampling distribution is 3.

σ x̄ = √∑ 𝐏(𝐱̄̄) • ( 𝐱̄̄ − µ )𝟐
= √𝟑
= 1.73
And the standard deviation of the sampling distribution is 1.73.

Let us summarize what we have done for the preceding examples by comparing the means
and variances of the population and the sampling distribution of the means.

In EXAMPLE 1 In EXAMPLE 2

Sampling Sampling
Distribution of Distribution of
Population Population
the Sample the Sample
(N=5) (N=3)
Means Means
(n=2) (n=2)

MEAN µ = 3 µx̄ = 3 µ = 5 µx̄ = 5

VARIANCE σ2 = 2 σ2 x̄ = 0.75 σ2 = 6 σ2 x̄ = 3
STANDARD
σ = 1.41 σ2 x̄ = 0.87 σ = 2.45 σ2 x̄ = 1.73
DEVIATION

NOTE:

Observe that the mean of the sampling distribution of the sample means is always equal to the
mean of the population. And the variance of the sampling distribution of the sample means is
smaller than the population distribution

73
Example 3: SAMPLING DISTRIBUTION OF THE SAMPLE MEANS FROM AN INFINITE POPULATION

A population has a mean of 60 and a standard deviation of 5. A random sample of 16 measurements


is drawn from this population. Describe the sampling distribution of the sample means by computing
its mean and standard deviation. We shall assume that the population is infinite.

STEPS SOLUTION

1.Identify the given information. Here µ = 60 , σ = 5 , and n = 16

µx̄ = µ
2. Find the mean of the sampling distribution. = 60
Use the Hence, the mean of the sampling distribution
property that µx̄ = µ of the
sample means is 60.

𝛔
3. Find the standard deviation of the sampling σ x̄ =
𝛔 √𝐧
distribution. Use the property that σ x̄ = 𝟓
√𝐧 =
√𝟏𝟔
𝟓
= or 1.25
𝟒
Hence the standard deviation of the sampling
distribution is 1.25.

Example 4: Sampling Distribution of the Sample Means for Normal Population When the Variance is
Unknown

A sample of size 9 is drawn from a normal population having a mean µ = 8 and the
standard deviation σ = 5. Suppose that the resulting sample is 11, 5, 7, 11, 10, 13, 13, 11, 9.
1. What is the sampling error based on this sample?
2. Determine the mean and standard deviation of the sampling distribution of sample means for samples
having the same size.

SOLUTION:
1. Using the provided data, the sample mean can be computed as follows:
𝟏𝟏 + 𝟓 + 𝟕 + 𝟏𝟏 + 𝟏𝟎 + 𝟏𝟑 + 𝟏𝟑 + 𝟏𝟏 + 𝟗
x̄ = = 10
𝟗
The sampling error is the difference between this value and the corresponding population
mean,

that is, 10 – 8 = 2

2. Our population has mean µ = 8 and and standard deviation σ = 5. Using the previous
generalization, the

sampling distribution of the sample means of samples with size 9 has a mean µx̄ = µ = 8 and
a standard
𝛔 𝟓 𝟓
deviation σ x̄ = = =
√𝐧 √𝟗 𝟑

74
Example 5: Sampling Distribution of the Sample Means for Normal Population When the Variance
is Known

A sample is drawn from a normal population with µ = 78.3 and σ = 5.6. How is the variance of the
sample mean affected when the sample size is

1. increased from 36 to 64 ?
2. decreased from 225 to 144 ?

SOLUTION: Since the population has standard deviation σ = 5.6, its variance is σ2 = ( 5.62 )2 =
31.36
31.36 31.36
1. The variance of the sample mean is decreased from ≈ 0.87 to ≈ 0.49
36 64
31.36 31.36
2. The variance of the sample mean is decreased from ≈ 0.14 to ≈ 0.22
225 144

III. ACTIVITIES

ACTIVITY 1: FIGURE IT OUT!


Choose the letter that corresponds to your answer.

________ 1. The weekly allowance of all the students in Malabog National High School has an average
of Php 50 and the standard deviation of Php 5. A random sample of 100 students was
asked their weekly allowance. What is the standard error of the sampling distribution of
the mean weekly allowance of students in this school?
5
a. 100 b. 5 c. 50 d.
√100
________ 2. For the scenario in item 2, the mean of the sampling distribution of the mean weekly
allowance of the students is ________.
10
a. 100 b. 5 c. 50 d.
√50
________ 3. Suppose the sampling error is zero. Which of the following must be true?
a. There is an error in the computation of the sample mean.
b. The sample statistic and the population parameter are proportional.
c. The sample statistic and the population parameter are the same.
d. None of the above.
________ 4. If a population has a mean of 5.7, what is the mean of the sampling distributions of its
means?
a. -5.7 b. 5.7 c. 0 d. Cannot be Determined

________ 5. How many possible samples of size 2 are there from the population consisting of the
values 10, 20, 40, and 70, if the sample is to be taken without replacement?
a. 4 b. 6 c. 8 d. 12

ACTIVITY 2: COMPLETE & DESCRIBE ME!


Directions: Describe the sampling distribution of the sample means of scores in the
Summative Test in Statistics. Complete the table below and compute for the mean, variance,
and standard deviation.

x̄ P ( x̄ ) x̄ • P ( x̄ ) x̄ - µ x̄ (x̄ - µ x̄ ) 2 P ( x̄ ) • (x̄ - µ x̄ ) 2
2
3
10
75
2
6
10
3
9
10
1
12
10
2
15
10
∑ 𝒙̄ • 𝑷 ( 𝒙̄ ) = ____ ∑ P(x̄ ) • ( x̄ − µx̄ )2 = ______

MEAN ( µ x̄ ) VARIANCE ( σ2 x̄ ) STANDARD DEVIATION ( σ x̄ )

IV. REFERENCES
Bataller, Ramil T. (2016). Statistics and Probability. Manila: Salesiana Books by Don Bosco Press,Inc.
Belecina, R., Baccay, E., & Mateo, E. (2016). Statistics and Probability. Manila: Rex Book Store, Inc.
Reyes, Maria Angeli T. (2017). Statistics and Probability. Manila: C & E Publishing, Inc.
https://www.slideshare.net/sirgibey/cabt-shs-statistics-probability-mean-and-variance-of-
sampling-distributions-of-
samplemeans?from_m_app=android&fbclid=IwAR1Sm1liVb1v2wqFmJUZim7WGQbTzCSQ1waKZIU
npmxKagLT3FcfMc3Z8ls

76
77
Writer
MARGIE C. MACATINGRAO
Prepared by:
ACTIVITY 1: FIGURE IT OUT!
1. a 4. b
2. c 5. b
3. a
ACTIVITY 2: COMPLETE & FIND ME!
x̄ P ( x̄ ) x̄ • P ( x̄ ) x̄ - µ x̄ (x̄ - µ x̄ ) 2 P ( x̄ ) • (x̄ - µ x̄ ) 2
2
3 0.6 -5.7 32.49 6.498 or 6.50
10
2
6 1.2 -2.7 7.29 1.458 or 1.46
10
3
9 2.7 0.3 0.09 0.027 or 0.03
10
1
12 1.2 3.3 10.89 1.089 or 1.09
10
2
15 3 6.3 39.69 7.938 or 7.94
10
2
∑ P(x̄ ) • ൫ x̄ − µx̄ ൯
∑ 𝒙̄ • 𝑷 ( 𝒙̄ )
= 17.02
= 8.7
MEAN ( µ x̄ ) VARIANCE ( σ2 x̄ ) STANDARD DEVIATION ( σ x̄ )
µ x̄ = ∑ 𝒙̄ • 𝑷 ( 𝒙̄ ) σ2 x̄ = ∑ 𝐏(𝐱̄̄) • ( 𝐱̄̄ − σ x̄ = √∑ 𝐏(𝐱̄̄) • ( 𝐱̄̄ − µ )𝟐
= 8.7 µ𝐱̄̄ )𝟐 = √𝟏𝟕. 𝟎𝟐
= 17.02 = 4.13
Therefore, the mean of Therefore, the standard
Therefore, the variance of deviation of the sampling
the sampling distribution
the sampling distribution is distribution is 4.13.
of the sample means is
17.02.
8.7.
Answer Key
STATISTICS AND PROBABILITY

Name of Learner: _____________________________ Section: _________________


Grade Level: _________________________________ Date: ___________________

Q3_W7&8_LAS13
SAMPLING DISTRIBUTION OF THE SAMPLE MEAN USING CENTRAL LIMIT THEOREM

I. BACKGROUND INFORMATION FOR LEARNERS

• The Central Limit Theorem states that as the sample size n increases without limit, the shape
of the distribution of the sample means taken with replacement from a population with mean
µ and standard deviation 𝝈 will approach a normal distribution. This distribution will have a
mean µ and a standard deviation 𝛔/√ 𝐧.
• A sampling distribution of sample means is a distribution using the means computed from all possible
random samples of a specific size taken from a population.

• Properties of the Distribution of Sample Means


1. The mean of the sample means (𝜇𝑥̅ ) will be the same as the population mean (µ).
2. The standard deviation of the sample means will be smaller than the standard deviation of
the population(𝜎𝑥̅ < 𝜎), and it will be equal to the population standard deviation divided by
the square root of the sample size ( 𝜎𝑥̅ =).

II. LEARNING COMPETENCIES


• Illustrates the Central Limit Theorem (M11/12SP-IIIe-2)
• Defines the sampling distribution of the sample mean using the Central Limit Theorem
(M11/12SP-III-3)

III. ACTIVITIES

Activity 1. Let’s Discover


Read the situation and answer the given questions.
Suppose a teacher gave a 4-item quiz to all his/her class with a total of four hundred students.
Assumed that each correct answer was given 2 points and no one in the class got a zero score. After
recording the teacher found out that 100 students got 2 points, 100 students got 4 points, 100
students got 6 points, and another 100 students got 8 points.

1. What is the mean (𝜇) score of the class?


a. 2 b. 3 c. 4 d. 5
2. What is the Standard Deviation of the students’ score? Use the formula below.
100(2 − 𝜇)2 + 100(4 − 𝜇)2 + 100(6 − 𝜇)2 + 100(8 − 𝜇)2
𝜎=√
400

100[(2 − 𝜇)2 + (4 − 𝜇)2 + (6 − 𝜇)2 + (8 − 𝜇)2 ]


𝜎=√
100(4)

(2 − 𝜇)2 + (4 − 𝜇)2 + (6 − 𝜇)2 + (8 − 𝜇)2


𝜎=√
4

a. 2 .236 b. 3 c. 1.581 d. 5
78
3. If you make a histogram of the students’ score, what would it look like?
a b c d

Suppose that the teacher who gave the 4-item quiz is teaching you statistics and he/she wants
you to take the mean and the standard deviation of the class in a short period of time without giving
you the summary of the result. Since you are in a hurry you just took few samples out of quiz scores.
Now, if all samples of size 2 are taken with replacement and the mean of each sample is found as
shown below.
Sample Mean Sample Mean
2,2 2 6,2 4
2,4 3 6,4 5
2,6 4 6,6 6
2,8 5 6,8 7
4,2 3 8,2 5
4,4 4 8,4 6
4,6 5 8,6 7
4,8 6 8,8 8
4. What is the mean of the sample mean?
a. 2 b. 3 c. 4 d. 5
5. What is the standard deviation of the sample mean?
a. 2 .236 b. 3 c. 1.581 d. 5
6. The mean of the population is _____ to the mean of the sample mean
a. not equal c. equal
b. greater than d. less than
7. The population standard deviation is ______ to the standard deviation of the sample mean
a. not equal c. equal
b. greater than d. less than
8. Based on the figure in activity 1 the dispersion (width) of the sample mean ______ is than the
population distribution
a. shorter c. wider
b. taller d. narrower
9. If a population is not normally distributed, the distribution of the sample means for a given
sample size n will ____________.
a. be positively skewed
b. be negatively skewed
c. takes the same shape as the population
d. approach a normal distribution as n increases
10. The standard deviation of the sampling distribution of the sample means according to the
Central Limit Theorem is
a. exactly equal to the standard deviation
b. close to the population standard deviation if the sample size is large
c. equal to the population standard deviation divided by the square root of the sample size
d. cannot be determined
79
Activity 2. LOOK AT ME
Observe the given figure and answer the given questions

NOTE: the line in the middle of each graph represents the mean of the population.

Population A B C D

Histogram of the
Population

Histogram of the
Sampling Distribution
when the sample size is
equal to 2

Histogram of the
Sampling Distribution
when the sample size is
equal to 6

Histogram of the
Sampling Distribution
when the sample size is
equal to 30

The figure above is based on the page 227 of the book Basic Statistics for Business and Economics by D.A. Lind,
W.G. Marchal, and S.A. Wathen (McGraw-Hill, 2006)

1. The graph of Population A is ____________.


a. Symmetric, Unimodal c. Symmetric, Bimodal
b. Uniform d. Rectangular

2. The graph of Population C is ____________.


a. Symmetric, Unimodal c. Symmetric, Bimodal
b. Uniform d. Rectangular

3. The shape of a Normal Distribution is like a ________.


a. Ghost c. Rectangle
b. Bell d. Triangle

4. Based on the figure as the value of sample size (n) gets ______, the curves approach a normal
distribution.
a. Smaller c. nearer to zero
b. Larger d. fatter

80
5. The graph population A and Population C are both symmetrical (but not normal). At what
value of n did the shape of the sampling distribution looks normal?
a. 2 b. 6 c. 4 d. 3

6. Population D is skewed to the right, at what value of n did the shape of the sampling
distribution looks normal?
a. 4 b. 6 c. 30 d. 2

7. At what value of n did all the graph of the sampling distribution look similar?
a. 2
b. 6
c. 4
d. 30

8-10. We can conclude that regardless of the initial __(8)__ of the population distribution, if
samples of size n are randomly selected from a population, the sampling distribution of the
sampling means will __(9)__ a normal distribution as the sample size n gets__(10)__ .

a. shape c. equal
b. approach d. large

IV. REFLECTION
What is the importance of the use of CLT in our daily life?

___________________________________________________________________________
__________________________________________________________________________________

Why is it useful in solving problems involving sampling?


___________________________________________________________________________
__________________________________________________________________________________

V. REFERENCES

Bluman, A. G. (2009). Elementary Statistics A Step by Step Approach. New York: McGraw-Hill.

Chan Shio, C. O., & Reyes, M. T. (2017). Statistics & Probability for Senior High School. Quezon City: C
& E Publishing, Inc.
Lind, D. A., Marchal, W. G., & Wathen, S. A. (2006). Basic Statistics for Business and Economics. New
York: McGraw-Hill.

81
82
Writer
Giovani G. Naag, ABE
Prepared by:
Answer Key
Activity 1.
1. C
2. B
3. B
4. B
5. C
6. C
7. D
8. A
9. B
10. D
Activity 2.
1. D
2. A
3. B
4. D
5. C
6. C
7. B
8. D
9. D
10. C
STATISTICS AND PROBABILITY

Name of Learner: _____________________________ Section: _________________


Grade Level: _________________________________ Date: ___________________

Q3_W7&8_LAS14
SOLVING PROBLEMS INVOLVING SAMPLING DISTRIBUTIONS OF THE SAMPLE MEAN

I. BACKGROUND INFORMATION FOR LEARNERS

A theoretical probability distribution of sample means that would be obtained by drawing from the
population all possible samples of the same size. In other words, the distribution of all possible values
from all the random samples of given size n.

 The probability distribution of sample mean (hereafter, will be denoted as 𝑋̅) is called the sampling
distribution of the mean (also, referred to as the distribution of sample mean).
 Like 𝑋̅, we call sampling distribution of variance (denoted as 𝑆2).
 Using the values of 𝑋̅ and 𝑆2 for different random samples of a population, we are to make
inference on the parameters 𝜇 and 𝜎2 (of the population).

Population parameter: a numerical descriptive measure of a population.

(for example: μ, 𝝈, p (a population proportion); the numerical value of a population parameter is


usually not known)

Examples: μ = mean height of all students

p=proportion of residents who favor stricter curfew p̂ (or

p-hat) = the mean of all sample proportions

standard deviation=

Sample statistic: a numerical descriptive measure calculated from sample data. (e.g, x, s, ̄
p̂(sample proportion))

Example 1 of sampling distribution of p̂, the sample proportion

• If a coin is fair the probability of a head on any toss of the coin is p = 0.5 (p is the
population parameter)
• Imagine tossing this fair coin 4 times and calculating the proportion p̂ of the 4 tosses that
result in heads (note that p̂ = x/4, where x is the number of heads in 4 tosses).

• Objective: determine the sampling distribution of p̂, the proportion of heads in 4 tosses of
a fair coin.

83
0 1 2 3 4
4 4 4 4 4
Possible values of: pˆ = 0, =.25, =.50, =.75, =1

There are 24 = 16 equally likely possible outcomes

(1 = head, 0 = tail)

(1,1,1,1) (1,1,1,0) (1,1,0,1) (1,0,1,1)

(0,1,1,1) (1,1,0,0) (1,0,1,0) (1,0,0,1)

(0,1,1,0) (0,1,0,1) (0,0,1,1) (1,0,0,0)

(0,1,0,0) (0,0,1,0) (0,0,0,1) (0,0,0,0)

p̂ 0.0 0.25 0.50 0.75


(0 heads) (1 head) (2 heads) (3 heads)

P(p) 1/16= 4/16= 6/16= 4/16=


0.0625 0.25 0.375 0.25

II. LEARNING COMPETENCY


• Solves problems involving sampling distributions of the sample mean. M11SP/IIIe-f-1

84
III. ACTIVITIES
Activity 1 LET’S PLAY!

Consider five identical balls numbered and weighting as 1, 2, 3, 4


and 5. Consider an experiment consisting of drawing two balls,
replacing the first before drawing the second, and then computing the
mean of the values of the two balls.

1. Following the given set of data, complete the table below by


providing their means and mass functions.
2. Draw the graph.

𝑿̅

𝒇(𝑿̅)

Activity 2: ONE, TWO, THREE, VOTE!

1. A school polling organization polls 100 randomly selected registered student voters to estimate the
proportion of a large population that intends to vote for Candidate X in an upcoming election.

Although it is not known by the polling organization, the actual proportion of the population that prefers
Candidate X is .55.

(a) Give the numerical value of the mean of the sampling


distribution of pˆ.

(b) Calculate the standard deviation of the sampling


distribution of pˆ.

(c) If the proportion of the population that prefers Candidate X


is .55, would a sample proportion value of .70 be considered
unusual?

Reflection
1. What have you learned from the lesson?
______________________________________________________________________________
______________________________________________________________________________
______________________________________________________________________________
2. Why do we need to apply sampling distribution in most of our daily tasks?

______________________________________________________________________________
______________________________________________________________________________
______________________________________________________________________________
3. What is the role of sampling distribution in solving both simple and complex word problems?

85
______________________________________________________________________________
______________________________________________________________________________
______________________________________________________________________________

IV. REFERENCES

• The Sampling Distribution of the Sample Mean. (2021, February 25). Retrieved March 12, 2021,
from https://chem.libretexts.org/@go/page/570
• Lind, D.A., Marchal, W.G. and Wathen, S.A. Basic Statistics for Business and Economics. 5th ed.
McGraw-Hill. 2006.
• Bluman, A. G. Elementary Statistics step by step approach. 7th Ed. McGraw-Hill. 2009.

86
87
Writer
Albert Corbilla
Prepared by:
c) Yes, a sample proportion of .70 would be considered unusual. It is
roughly 3 standard deviations above the mean: .55 + 3(.05) = .70.
ANSWER KEY
STATISTICS AND PROBABILITY

Name of Learner: _____________________________ Section: _________________


Grade Level: _________________________________ Date: ___________________

Q3_W9_LAS15
ILLUSTRATING T-DISTRIBUTION

I. BACKGROUND INFORMATION FOR LEARNERS


t- distribution which is also known as Student’s t-distribution, is a probability distribution
which is utilized in estimating parameters of a certain population in case of the sample size is
small and/or the population variance or standard deviation is unknown. It is like with the z-
distribution that is bell-shaped and symmetric about the x- axis but flatter and more spread.

Take note that z-test is only used when the population standard deviation or variance
is known and/or the sample size is large enough. But for sample sizes which are small, sometimes
the population standard deviation or variance is unknown, rely on the t-distribution whose value
can be known using: 𝑥̅ − 𝜇
𝑡= 𝑠
√𝑛

where 𝑥 is the sample mean, 𝜇 is the population mean, s is the standard deviation of the
sample and n is the sample size.
In addition, there is a lot of different or family of t-distributions. Its form is determined by its
degrees of freedom. Degrees of freedom refers to the number of independent observations in
each set of data. The number of independent observations is sample size minus one or in
symbols, that is df= n-1 where df is the degrees of freedom and n is the sample size. Hence,
given a sample size of 9 will have a degree of freedom 8 and if given a degree of freedom of 14
will mean a sample size of 15. However, for some or other applications, degrees of freedom can
be calculated in a different way.

88
Moreover, some properties of t-distribution is that the mean distribution is equal to 0 and
its variance is greater than 1 although close to 1. T- distribution is the same as the standard
normal distribution given with infinite degrees of freedom.
With any statistic that have a bell- shaped distribution with is approximately
normal, the t-distribution can be used. This means that the population distribution must be
normal, symmetric, unimodal and no outliers. Also, the t-distribution can still be utilized for
moderately skewed population distribution given that it is unimodal, without outliers and size is
at least 40. Lastly, the t-distribution can be applied for a size greater than 40 and without outliers.
The t- distribution Table

89
To get the t- value, a t-distribution table is shown above which is consist of the degrees of
freedom (df) which is the numbers at the leftmost column, “α” which is some of the special areas
at the topmost row and the t-values which are located at the right of the degrees of freedom and
below “α”.
Identifying the t-value
To denote the t-value, it is conveniently written as t(𝛼 ,𝑑𝑓). So, if you are ask on the t-value
denoted by t(0.05 ,8) means the t-value corresponding to 𝛼 = 0.05 and df= 8. To look for
this value in the t-table, first locate the 𝛼 = 0.05 on the top row and then the df on the
leftmost column. The intersection of 𝛼 = 0.05 and df= 8 is 1.860. (See illustration
below)

df 𝛼 0.1 0.05
1 3.078 6.314
2 1.886
2.920
: :

8 1.397
1.860

II. LEARNING COMPETENCY

Illustrates t- distribution (M11/12SP-IIIg-2).

Examples:
Determine what is asked based on the given data below.

1. n= 14 𝛼 = 0.1 t(𝛼 ,𝑑𝑓) = _____________

2. df= 10 𝛼 = 0.025 t(𝛼 ,𝑑𝑓) = _____________

3. t(𝛼 ,𝑑𝑓)= 2.052 𝛼=0.025 n = __________________

4. Find the t-value when 𝜇 = 42, 𝑥 = 44, s= 5 and n=25

90
Solution
1. n= 14 𝛼 = 0.1 t( 𝛼 , 𝑑𝑓 ) = 1.350

If n= 14, so df = n-1= 14-1= 13

df 𝛼

0.1

1 3.078

2 1.886



13 1.350
1

2. df= 10 𝛼 = 0.025 t( 𝛼 , 𝑑𝑓 ) = 2.228

df 𝛼 0.1 0.05 0.025

1 3.078 6.314

2 1.886 2.920

⋮ ⋮ ⋮

10 1.372 1.812
3. t(𝛼 ,𝑑𝑓)= 2.052 𝛼=0.025 n = 28 2.228

df 𝛼 0.1 0.05 0.025

1 3.078 6.314

2 1.886 2.920

⋮ ⋮ ⋮

27
1.372 1.812
91
2.025
Since: df= 27

df= n –1

27= n – 1
27+1= n (Addition Property of Equality)

N = 28

4. Find the t-value when 𝜇 = 42, 𝑥 = 44, s= 5 and n=25.

𝑥̅ − 𝜇 44 − 42 3 3
𝑡(α , df) = 𝑠 = = = =3
5 5 1
√𝑛 √25 5

Guide Questions
1. When are you going to use the t-distribution?

2. What are the properties of t-distribution?

III. ACTIVITIES
Activity 1: What can I do?

Determine what is asked based on the given data below.

1. n= 20 𝛼 = 0.005 t( =____________
∝, 𝑑𝑓)

2. df= 28 𝛼 = 0.01 t( = ____________


∝, 𝑑𝑓)

3. Find the t-value when 𝛼= 18.5, 𝜇 = 19, s= 2.5 and n=16.

Rubric for scoring

DISTINGUISHED 4 PROFICIENT APPRENTICE 2 NOVICE


3 1

Uses a representation Uses a Uses a Uses a


that is unusual in its representation representation that representation
mathematical precision that clearly gives some that gives little
Uses
depicts the important or no significant
Representations
problem information about information
the problem about the
problem

Correct solution of Correct solution Copying error, No answer or


problem and made a computational error, wrong answer
general rule about the partial answer for based upon an
Answers the solution or extended the problem with inappropriate
Problem solution to a more multiple answers, no plan
complicated solution answer statement,
answer labeled
incorrectly

92
IV. REFLECTION
t-Distribution value enables one to determine its location in the bell-shaped distribution and
use as basis for making decisions. How do you find the t-value? When do you use t-value? What
about z-value?

V. REFERENCES
Cervantes, Irl John M. Q3-Module 4-Estimation of Parameters, pp 1-8

ANSWER KEY

2. 2.467

1. 1.729

Prepared by

HILDA C. REMENDADO
Writer

93
STATISTICS AND PROBABILITY

Name of Learner: _____________________________ Section: _________________


Grade Level: _________________________________ Date: ___________________

Q3_W9_LAS16
LEARNING ACTIVITY SHEET
IDENTIFYING PERCENTILE RANK AND t-DISTRIBUTION

I. BACKGROUND INFORMATION FOR LEARNERS


Percentile is one of the measurements in statistics which tells the value below in which an
observations’ percentage in a set of observations falls. For example, you score 70 in an exam, and it
was mentioned that you the 85th percentile, it means that 85% of the scores are below you and 15%
of the scores are above you. In addition, in finding the percentile for a t- distribution, t-table can be
utilized as it is a number on a statistical distribution whose less- than the probability is the given
percentage. So, if you are asking on the 80th percentile of the t-distribution with respect to its degrees
of freedom, that refers to the value whose left tail is less than probability is 80% or 0.8 and whose
right tail or greater than probability is 20% or 0.2

II. LEARNING COMPETENCY


Identifies percentiles using t-distribution (M11/12SP-IIIg-5)

Directions
Study the solutions of exercises below, answer Activities part by showing your solution in a
separate sheet and submit to your teacher.

Examples: Determine what is asked based on the given data below.


1. n= 14, Percentile = 95th, 𝑡(𝛼, 𝑑𝑓) = ____________________
2. df= 10, Percentile = 90th, 𝑡(𝛼, 𝑑𝑓) = ____________________

Solution:
1. n= 14, Percentile = 95th, 𝑡(𝛼,𝑑𝑓)= 1.771
To find the value of 95th percentile, identify first the degrees of freedom.
df = n - 1 = 14 - 1 = 13
To solve for 95th percentile, we need to understand first its implication which is to get the t-value
that is less than the probability 0.95 and the right tail probability that is 0.05. Based on the table,
the 95th percentile is 1.771.
t(df 𝛼 0.05 1 6.314 2 2.920 ⋮ ⋮ 13 1.771
2. df = 10, Percentile= 90th
𝑡(𝛼,𝑑𝑓) = 1.372 , since the degrees of freedom is given already, we can now identify the 90th
percentile which is 1.372. df 𝛼 0.1 1 3.078 2 1.886 ⋮ ⋮ 10 1.372

III. ACTIVITIES

Determine what is asked based on the given data below. Show solutions in a separate sheet of
paper.
1. n= 15 Percentile= 99.5th 𝑡(𝛼,𝑑𝑓)=
2. df= 25 Percentile= 97.5th 𝑡(𝛼,𝑑𝑓)=
3. n= 20 Percentile= 99.5th 𝑡(𝛼,𝑑𝑓)=
4. df= 31 Percentile= 97.5th 𝑡(𝛼,𝑑𝑓)=
94
5. n= 5 Percentile= 90th 𝑡(𝛼,𝑑𝑓)=

Guide Questions
1. Why is percentile important?
2. What is/are the disadvantages of percentile?
Rubric for scoring
DISTINGUISHED PROFICIENT APPRENTICE NOVICE
4 3 2 1

Uses a Uses a Uses a Uses a


representation that representation representation representation
is unusual in its that clearly that gives that gives little
Uses mathematical depicts the some or no
Representations precision problem important significant
information information
about the about the
problem problem

Correct solution of Correct Copying error, No answer or


problem and made solution computational wrong answer
a general rule error, partial based upon an
about the solution answer for inappropriate
or extended the problem with plan
Answers the
solution to a more multiple
Problem
complicated answers, no
solution answer
statement,
answer labeled
incorrectly

IV. Reflection
Your percentile rank is your location in an area where you are above or at the right. Strive
more to be in higher spot for self-improvement. Use statistics to compute your percentile rank.
What application or impact does it bring your life?

V. REFERENCES
Cervantes, Irl John M. Q3-Module 4-Estimation of Parameters, pp 1-8

Answer Key
1.533 5.
1.96 4.
2.861 3.
2.060 2.
2.997 1.

Prepared by
HILDA C. REMENDADO
Writer
95
STATISTICS AND PROBABILITY

Name of Learner: _____________________________ Section: _________________


Grade Level: _________________________________ Date: ___________________

Q3_W10_LAS17
COMPUTING THE LENGTH OF CONFIDENCE INTERVAL

I. BACKGROUND INFORMATION FOR LEARNERS


• A confidence interval is a specific interval estimate of a parameter determined by using data
obtained from a sample and by using the specific confidence level of the estimate. It is simply
a way to measure how well your sample represents the population you are studying.
• The confidence level is the probability that the confidence interval includes the true mean
value within a population.
𝐶𝑜𝑛𝑓𝑖𝑑𝑒𝑛𝑐𝑒 𝑙𝑒𝑣𝑒𝑙 = (1 − 𝑎)100%
• Length of Confidence Interval refers to the absolute difference between the upper confidence
limit and lower confidence limit.

IMPORTANT FORMULAS:
o Given the upper and lower confidence limit
𝐿𝐶𝐼 = |𝑈𝐶𝐿 − 𝐿𝐶𝐿| = |𝐿𝐶𝐿 − 𝑈𝐶𝐿|
𝑳𝑪𝑰 = 𝑼𝑪𝑳 − 𝑳𝑪𝑳
o Confidence interval of population mean (µ) with known variance
if n≥30 (use the z-distribution table)
𝜎 𝜎
𝑋̅ − 𝑍𝑎⁄2 < 𝜇 < 𝑋̅ + 𝑍𝑎⁄2
√𝑛 √𝑛
𝝈
𝑳𝑪𝑰 = 𝟐 𝒁𝒂⁄𝟐
√𝒏
o Confidence interval of population mean (µ) with known variance
if n<30 (use the t-distribution table)
𝜎 𝜎
𝑋̅ − 𝑡𝑎⁄2 < 𝜇 < 𝑋̅ + 𝑡𝑎⁄2
√𝑛 √𝑛
𝝈
𝑳𝑪𝑰 = 𝟐 𝒕𝒂⁄𝟐
√𝒏
Where:
LCI- length of confidence interval
UCI- upper confidence limit
LCL- lower confidence limit
𝑍𝑎⁄ = z value from the standard normal distribution
2
curve
𝑡𝑎⁄ = t value with df=n-1
2
σ = standard deviation
n= sample size

96
Table1: Z-Scores for Commonly used Confidence Level
DESIRED CONFIDENCE LEVEL (Α) 𝒁𝜶⁄
𝟐
90% 1.65
95% 1.96
99% 2.58
In this learning activity sheet, the learners will be able to know how to identify and compute for the
length of confidence interval.

Examples:

1. Find the length of the confidence interval 0.275 < 𝑝 < 0.463

STEPS SOLUTION
1. Determine the given UCL= 0.463
LCL= 0.275
2. Use the formula for Length of LCI= UCL-LCL
Confidence Interval
3. Substitute the given value, then LCI= 0.463-0.275
perform the indicated operation. LCI= 0.188
Thus, the length of confidence interval is equal to 0.188

2. Find the length of confidence interval given the following data:

σ = 2.35 n= 250 confidence level= 99%

STEPS SOLUTION
1. Determine the given σ = 2.35, n= 250, confidence level= 99%
2. Determine the 𝑎 in (1- (1 − 𝑎)100% = 99%
a)100% confidence 1 − 𝑎 = 0.99
level, then find 𝑍𝑎⁄2 𝑎 = 0.01
𝑎 0.01
= = 0.005
2 2
Subtract 0.005 from 0.5 (which is half the area of the
Standard Normal Curve)
0.500 − 0.005 = 0.495
Using the Standard Normal Table,
𝑍𝑎⁄ = 2.58
2
3. Substitute the values in 𝜎
𝐿𝐶𝐼 = 2 𝑍𝑎⁄2
the formula and √𝑛
compute. 2.35
𝜎 𝐿𝐶𝐼 = 2 (2.58)
𝐿𝐶𝐼 = 2 𝑍𝑎⁄2 √250
√𝑛 𝐿𝐶𝐼 = 0.7669
Thus, the length of confidence interval is equal to 0.7669

97
3. Find the length of confidence interval given the following data:

σ = 3.25 n= 17 confidence level= 95%

STEPS SOLUTION
1. Determine the given σ = 3.25, n= 17, confidence level= 95%
2. Find the degree of 𝑑𝑓 = 𝑛 − 1
freedom 𝑑𝑓 𝑑𝑓 = 17 − 1
𝑑𝑓 = 16
3. Determine the 𝑎 in (1- (1 − 𝑎)100% = 95%
a)100% confidence 1 − 𝑎 = 0.95
level, then find 𝑡𝑎⁄2 𝑎 = 0.05
𝑎 0.05
= = 0.025
2 2

Using the t distribution critical values (t-table) at 𝑑𝑓 =


𝑎
16 and = 0.025
2
𝑡𝑎⁄ = 2.120
2
4. Substitute the values in 𝜎
𝐿𝐶𝐼 = 2 𝑡𝑎⁄2
the formula and √𝑛
compute. 3.25
𝜎 𝐿𝐶𝐼 = 2 (2.120)
𝐿𝐶𝐼 = 2 𝑍𝑎⁄2 for √17
√𝑛 𝐿𝐶𝐼 = 3.3421
n<30

Thus, the length of confidence interval is equal to 3.3421

II. LEARNING COMPETENCIES


• Identifies the length of a confidence interval (M11/12SP-IIIj-1).
• Computes for the length of confidence interval (M11/12SP-IIIj-1).

98
III. ACTIVITIES

DIRECTIONS: Read and understand the directions in each activity. If you have any question, feel
free to message your teacher for clarification/assistance.

ACTIVITY 1: WHO IS HE?

Directions: Find out the identity of the man who first introduced the modern concept of confidence
interval into statistical hypothesis testing by identifying the length of the confidence interval and use
the corresponding letter to complete the table provided below. Some letter/s will be left unused.
Show your solutions.

Y V
0.368 < 𝑝 < 0.567 0.672 < 𝑝 < 0.921

J U
UCL = 0.543 UCL =0.954
LCL =0.398 LCL =0.564

Murphy, M. (2017, April 13). Amstat News: The Membership Magazine of the
American Statistical Association. Retrieved from
https://magazine.amstat.org/blog/2017/04/13/sih-neyman/

E L D N
0.275 < 𝑝 < 0.463 UCL = 0.910 0.275 < 𝑝 < 0.463 UCL =0.678
LCL =0.822 LCL =0.392

M W R A
0.475 < 𝑝 < 0.663 UCL =0.467 0.329 < 𝑝 < 0.563 0.426 < 𝑝 < 0.663
LCL =0.242

Name
Answer 0.145 0.188 0.234 0.249 0.199 0.286 0.188 0.225 0.198 0.237 0.286

99
ACTIVITY 2: CRACK THE CODE AND OPEN THE LOCK!

Directions: In this Crack the Code puzzle, you are given three digits, some hints are already given
regarding the correctness of these digits. Your challenge is to decode the correct 3 digits code by
computing the length of the confidence interval and then open the lock. Show your solutions.

So, are you ready to crack the code?

One number is correct and


2 1 4 well placed

7 6 2 Nothing is correct

Two numbers are correct but


5 3 0 wrongly placed
? ? ?
Find the length of the confidence interval given the following data:

1 5
σ = 3.72 n= 150 confidence level= 99% Fifteen individuals were asked how long they
slept at night. The mean time was 7.1 hours, and
the standard deviation was 0.78 hour. If the 90%
confidence interval was adopted, find the length
of the confidence interval.

2 6
σ = 1.20 n= 17 confidence level= 90% σ = 5.19 n=530 confidence level= 99%

3 7
σ = 3.25 n= 29 confidence level= 95% Consider a random variable that is normally
distributed with a standard deviation of 5 and a
4 sample size of 230. Find the length of confidence
σ = 6.5 n= 35 confidence level= 95% interval at a confidence level of 95%

Code
Answer 2.4720 1.5673 0.7093

100
III. REFLECTION
1. Cite an example where confidence interval is used in real life.
___________________________________________________________________________
___________________________________________________________________________
_______________________________________________________________

2. Which do you think is the best confidence interval to use? Why?


___________________________________________________________________________
___________________________________________________________________________
_______________________________________________________________

IV. REFERENCES
Bluman, A. G. (2009). Elementary Statistics: A step by Step Approach (7th Edition). In A. G. Bluman,
Elementary Statistics: A step by Step Approach (7th Edition) (pp. 356-392). New York City:
McGraw-Hill Companies, Inc.

Lisa Sullivan, P. (n.d.). Confidence Intervals. Retrieved from Boston University School of Public
Health: https://sphweb.bumc.bu.edu/otlt/mph-
modules/bs/bs704_confidence_intervals/bs704_confidence_intervals_print.html

Murphy, M. (2017, April 13). Amstat News: The Membership Magazine of the American Statistical
Association. Retrieved from https://magazine.amstat.org/blog/2017/04/13/sih-neyman/

Rene R. Belecina, E. S. (2016). RBS Statistics and Probability (First Edition). In E. S. Rene R. Belecina,
RBS Statistics and Probability (First Edition) (pp. 200-210). 856 Nicanor Reyes
Sr.St.,Sampaloc, Manila: Rex Book Store, Inc (RBSI).

101
102
Writer
MARY FRANCIA S. RICO, RChE
Prepared by:
Activity 1: WHO IS HE?
Y V J U
𝐿𝐶𝐼 = 0.199 𝐿𝐶𝐼 = 0.249 𝐿𝐶𝐼 = 0.145 𝐿𝐶𝐼 = 0.390
E L D N
𝐿𝐶𝐼 = 0.188 𝐿𝐶𝐼 = 0.088 𝐿𝐶𝐼 = 0.188 𝐿𝐶𝐼 = 0.286
M W R A
𝐿𝐶𝐼 = 0.198 𝐿𝐶𝐼 = 0.225 𝐿𝐶𝐼 = 0.234 𝐿𝐶𝐼 = 0.237
Name J E R V Y N E W M A N
Answer 0.145 0.188 0.234 0.249 0.199 0.286 0.188 0.225 0.198 0.237 0.286
Activity 2: CRACK THE CODE AND OPEN THE LOCK!
1 5
𝐿𝐶𝐼 = 1.5673 𝐿𝐶𝐼 = 0.8249
2 6
𝐿𝐶𝐼 = 1.0163 𝐿𝐶𝐼 = 1.1633
3 7
𝐿𝐶𝐼 = 2.4720 𝐿𝐶𝐼 = 1.2924
4
𝐿𝐶𝐼 = 4.3069
Code 3 1 5
Answer 2.4720 1.5673 0.7093
ANSWER KEY
STATISTICS AND PROBABILITY

Name of Learner: _____________________________ Section: _________________


Grade Level: _________________________________ Date: ___________________

Q3_W10_LAS18
DETERMINING THE SAMPLE SIZE DETERMINATION USING
THE LENGTH OF CONFIDENCE LEVEL INTERVAL

I. BACKGROUND INFORMATION FOR LEARNERS


Two things to remember when we decide on the quality of the sample size we need:
confidence and the narrowness of the interval.
The computing formula in determining sample size is derived from the following formula:

Deriving Formula 1 Margin of Error


𝝈
𝑬 = 𝒁𝒂⁄𝟐
√𝒏

Formula in Determining Minimum Sample Size Needed when Estimating the Population
Mean:
𝒁𝒂⁄ ∗ 𝝈 𝟐
𝒏=( 𝟐 )
𝑬

Note: When determining sample size, we always round up the resulting value to the next
whole number.

Example 1:
Teacher France wants to conduct a survey about the average number of students in
her school who wants online class instead of distance modular learning. If she plans to use
99% confidence level, 0.5 as the margin of error, and a standard deviation of 5. How many
sample sizes does she need for the survey?

STEPS SOLUTIONS
Step 1: Determine the Confidence Level=99%; E=0.5; σ = 5
given.

Step 2: Determine the (1 − 𝑎)100% = 99%


confidence coefficient. 1 − 𝑎 = 0.99
𝑎 = 0.01
𝑎 0.01
= = 0.005
2 2

103
Subtract 0.005 from 0.5 (which is half the area of the
Standard Normal Curve)
0.500 − 0.005 = 0.495

Using the Standard Normal Table,

𝑍𝑎⁄ = 2.58
2

Step 3: Substitute the 𝑍𝑎⁄ ∗ 𝜎 2


values in the formula and 𝑛=( 2 )
𝐸
compute
(2.58)(5) 2
𝑛=[ ]
0.5
𝑛 = 665.65 𝑜𝑟 666
Thus, Teacher France needs a sample size of 666 for the
survey.

Deriving Formula 2 Margin of Error


̂ ̂𝒒
𝒑
𝑬 = 𝒁𝒂⁄𝟐 √
𝒏

Formula in Determining Minimum Sample Size Needed when Estimating the Population
Proportion:
𝒁𝒂⁄ 𝟐
𝒏=𝒑 ̂ ̂𝒒 ( 𝟐)
𝑬
𝒙
Where: ̂𝒑 = point estimate for the true population proportion, 𝒑̂=
𝒏
x = number of successes in the sample
̂𝒒 = point estimate for the population proportions of failures, ̂𝒒 =
̂
𝟏−𝒑

Note: This formula assumes that you have preliminary estimates for 𝑝̂ and ̂𝑞. If not, use
𝑝̂ = 0.5 and ̂𝑞 = 0.5

104
Example 2:
Susan wants to know with 95% confidence the proportion of households who likes to
use Detergent X. A previous survey showed that 42% like to use Detergent X. Susan likes to
be accurate within 2% of the true proportion. What sample size does Susan need?

STEPS SOLUTIONS
Step 1: Determine the Confidence Level=95%; E=2% or 0.02; proportion=42%
given.

Step 2: Determine the With 95% confidence level, a= .05


confidence coefficient. So,
𝑍𝑎⁄ = 1.96
2
Step 3: Determine 𝒑̂ and ̂𝒒. The given proportion is 42%
That is 𝑝̂ = 0.42.
So, ̂𝑞 = 1 − ̂𝑝 = 1 − 0.42 = 0.58
Step 4: Substitute the 𝑍𝑎⁄ 2
values in the formula and 𝑛 = 𝑝̂ 𝑞̂ ( 2)
compute 𝐸
1.96 2
𝑛 = (0.42)(0.58) ( )
0.02
𝑛 = (0.42)(0.58)(9604)
𝑛 = 2339.53 𝑜𝑟 2340
Thus, Susan needs a sample of 2340 respondents.

II. LEARNING COMPETENCIES

• Computes for an appropriate sample size using the length of the confidence
interval. (M11/12SP-IIIj-3)
• Solve problems involving sample size determination. (M11/12SP-IIIj-4)
III. ACTIVITIES
Directions/Instructions: Read and understand the directions in each activity. If you have any
question, feel free to message your teacher for clarification/assistance.
Activity 1: I CAN DO THIS!
Determine the sample size given the following data.

1. S=10 E= 5 Confidence level= 95%


2. S=3.30 E= 1.03 Confidence level= 99%
3. S= 8 E= 0.25 Confidence level= 90%
4. p̂ = 0.52 E= 0.18 95% confidence
5. p̂ = 0.38 E= 0.08 90% confidence

105
Activity 2: WHICH WAY TO GO?
Directions: In this maze, the path from the entrance to the exit is hidden by math problems.
Answer all the problems correctly and it will guide you to the end of the maze. Make a
mistake, and you will be led in the wrong direction. Worse, taking a wrong turn could reveal
a path of correct answers that eventually dead ends.
START HERE!!

The principal wants to use the


proportion of a population to A medical research team feels
determine the sample size needed to confident that a vaccine they have
interview regarding their thoughts developed will cure about 75% of
about the new school structure. She the patients suffering from COVID-
wants to be able to assert with a 350 19 virus. How large should the
probability 0.95 that her error will be at sample size be for the team to be
most 0.05. Similar polls in the past 98% confident that the sample
showed that 65% approved the new proportion of the cures is within 4%
structure. How large a sample does the of the proportion?
Principal need?

914 10

You are running a political campaign a


A statistic practitioner would like to nd wish to estimate, with 95% confide
estimate a population mean within nce, the population proportion of regi
stered voters who will vote for your ca
10 units with 99% confidence given
4161 ndidate. Your estimate must be accur
that the population standard ate within 3% of the population propo
deviation is 250. What sample size rtion. Find the minimum sample size
should be used? needed when no preliminary estimate
is available.

2150 1068

A travel agent wants to estimate the


proportion of vacationers who plan to
FINISH
travel outside the Philippines in the
next 12 months. A random sample of
150 vacationers revealed that 45 had
plans for foreign travel in that time
frame. Suppose (at the 95% confidence 505
level) you need to have a margin of
error no more than 4 percentage
points. How many vacationers would
you have to sample?

106
IV. REFLECTION
1. What happens to the length of confidence interval when the sample size increases?
_____________________________________________________________________
_____________________________________________________________________
_____________________________________________________________________

2. Does increasing sample size increase accuracy or precision? Explain your answer.
_____________________________________________________________________
_____________________________________________________________________
_____________________________________________________________________

V. REFERENCES

Bluman, A. G. (2009). Elementary Statistics: A step by Step Approach (7th Edition). In A. G.


Bluman, Elementary Statistics: A step by Step Approach (7th Edition) (pp. 356-392).
New York City: McGraw-Hill Companies, Inc.
Lisa Sullivan, P. (n.d.). Confidence Intervals. Retrieved from Boston University School of
Public Health: https://sphweb.bumc.bu.edu/otlt/mph-
modules/bs/bs704_confidence_intervals/bs704_confidence_intervals_print.html
Rene R. Belecina, E. S. (2016). RBS Statistics and Probability (First Edition). In E. S. Rene R.
Belecina, RBS Statistics and Probability (First Edition) (pp. 200-210). 856 Nicanor
Reyes Sr.St.,Sampaloc, Manila: Rex Book Store, Inc (RBSI).
Statistics Lesson 6.3 Notebook. (2017, May 03). Retrieved from
https://www.hanover.k12.in.us/cms/lib/IN01001361/Centricity/Domain/104/Stats%
206.3.pdf

107
108
Writer
MARY FRANCIA S. RICO, RChE
Prepared by:
Activity 1: I CAN DO THIS!
1. 16
2. 69
3. 15
4. 30
5. 100
Activity 2: WHICH WAY TO GO?
ANSWER KEY

You might also like